Sie sind auf Seite 1von 78

Matemática e Invariantes

1
2
Indice
1. Pregunta y Advertencia 4

2. El Engaño 5
2.1. Introducción . . . . . . . . . . . . . . . . . . . . . . . . . . . . . . . . 6
2.2. Primeros pasos . . . . . . . . . . . . . . . . . . . . . . . . . . . . . . 8
2.3. Luces . . . . . . . . . . . . . . . . . . . . . . . . . . . . . . . . . . . . 14
2.4. Piedras . . . . . . . . . . . . . . . . . . . . . . . . . . . . . . . . . . . 18
2.5. Fichas . . . . . . . . . . . . . . . . . . . . . . . . . . . . . . . . . . . 20
2.6. Un poco de Geometría Proyectiva . . . . . . . . . . . . . . . . . . . . 24
2.7. Cartas . . . . . . . . . . . . . . . . . . . . . . . . . . . . . . . . . . . 31
2.8. Rompecabezas . . . . . . . . . . . . . . . . . . . . . . . . . . . . . . . 36
2.9. Pizarrones, Tablas y Grupos . . . . . . . . . . . . . . . . . . . . . . . 39
2.10. Senku . . . . . . . . . . . . . . . . . . . . . . . . . . . . . . . . . . . 43
2.11. Ranas . . . . . . . . . . . . . . . . . . . . . . . . . . . . . . . . . . . 46
2.12. Monovariantes . . . . . . . . . . . . . . . . . . . . . . . . . . . . . . . 48
2.13. Un problema y 2 soluciones . . . . . . . . . . . . . . . . . . . . . . . 51
2.14. Diamante Azteca . . . . . . . . . . . . . . . . . . . . . . . . . . . . . 53
2.15. Pulgas . . . . . . . . . . . . . . . . . . . . . . . . . . . . . . . . . . . 59

3. Apéndices 61
3.1. Sumas infinitas . . . . . . . . . . . . . . . . . . . . . . . . . . . . . . 62
3.2. Sumas de potencias . . . . . . . . . . . . . . . . . . . . . . . . . . . . 65
3.3. Sucesiones recurrentes . . . . . . . . . . . . . . . . . . . . . . . . . . 67
3.4. Permutaciones . . . . . . . . . . . . . . . . . . . . . . . . . . . . . . . 69
3.5. Menelao . . . . . . . . . . . . . . . . . . . . . . . . . . . . . . . . . . 71

4. Problemas 75
4.1. Algunos problemas para pensar . . . . . . . . . . . . . . . . . . . . . 76

3
1. Pregunta y Advertencia

Advertencia: Cada solución que escribía despertaba el mismo miedo, que iban
a leer la solución antes de pensar el problema y así no sólo perderían la oportunidad de
pensar el problema sino que entenderían menos la solución. Por eso esta advertencia,
es mejor pensar un problema por algún tiempo y que no salga que leer su solución en
5 minutos y olvidarse de él. Piensen cada problema, no se apresuren a leer su solución.

Pregunta: “Hacer matemática no es cuestión de acumular ideas, trucos o teo-


remas para que cada vez que se nos presente un problema repasemos la lista hasta
dar con el teorema indicado.....ni siquiera es importante conocer toda clase de teo-
remas, trucos o técnicas....“ ¿Por qué entonces escribir algo que parece un manual
para resolver problemas usando invariantes? ¿No parece esto tratar de listar todos los
invariantes habidos y por haber?

4
2. El Engaño

5
2.1. Introducción

Supongamos que tenemos un objeto que va cambiando de acuerdo a ciertas reglas.


Nos podemos preguntar:

¿En qué puede transformarse?

¿Cuánto tardará en llegar a cierto estado?

¿Puede cambiar indefinidamente?

¿Puede transformarse en algo en particular?

Los invariantes nos ayudan a responder estas preguntas, la idea es mirar lo que
no cambia a medida que nuestro objeto va tranformándose. Veamos un ejemplo, el
problema es de la XI Olimpíada Nacional Argentina.

Se tienen 3 hormigas en los vértices de un cuadrado. En cada turno, una


hormiga se puede mover en dirección paralela a la recta que determinan
las otras 2. ¿Es posible que después de algunos turnos las hormigas ocupen
3 puntos medios de los lados del cuadrado?

Figura 1: La hormiga en C puede ir hasta D.

6
Solución: No, no es posible. Notemos que el área del triángulo que forman las 3
hormigas no varía con los turnos, pues cada vértice se mueve en forma paralela al lado
opuesto1 . Si el cuadrado inicial tenía área 1, entonces 3 vértices forman un triángulo
de área 12 y 3 puntos medios forman un triángulo con area 14 . Entonces no es posible
que las hormigas terminen en 3 puntos medios de los lados del cuadrado. ‡

Figura 2: El área nunca cambia

En este caso el objeto era la posición de las 3 hormigas y el “Invariante“ el área


del triángulo que formaban.

1
Lo que estamos usando es que si tenemos 2 triángulos ABC y ABD con AB//CD entonces
área(ABC) = área(ABD)

7
2.2. Primeros pasos
Para empezar, algunos ejemplos sencillos. En todos los problemas tenemos algo
que va cambiando de acuerdo a ciertas reglas, lo que tenemos que hacer es mirar con
cuidado y encontrar algo que no cambia.

• En el pizarrón hay 10 signos + y 5 signos −. En cada turno se debe


elegir dos de ellos, borrarlos y poner un + si eran iguales y un − si eran
distintos. Probar que después de 14 turnos quedará un signo − solamente.

Si pensamos a cada signo + como un 1 y a cada signo − como un −1, al borrar 2


números los debemos reemplazar por su producto, entonces el producto de todos los
números en el pizarrón no cambia. Como inicialmente el producto era 110 (−1)5 = −1,
entonces luego de 14 turnos cuando sólo queda un número, éste debe ser −1. ‡

• En el pizarrón hay escritos 2 números, en cada turno se borran los 2


números y si éstos eran a y b entonces se reemplazan por 2a − b y 2b − a.
Si inicialmente estaban escritos el 1458 y 1460. ¿Es posible que luego de
varios turnos los números en el pizarrón sean el 715 y el 1024?

Como 2a − b + 2b − a = a + b entonces la suma de los 2 números escritos en el


pizarrón nunca cambia. Al principio ésta es 1458 + 1460 = 2918, como 715 y 1024 no
suman 2918 entonces los números en el pizarrón nunca serán 715 y 1024. ‡

• Se tiene un tablero de 8 × 8 de ajedrez pintado de blanco y negro de la


forma usual. En cada turno se pueden intercambiar 2 columnas o 2 filas.
¿Es posible lograr que todas las casillas negras estén en la mitad izquierda
del tablero y las casillas blancas en la mitad derecha?

Notemos que la cantidad de casillas negras en cada columna nunca varía, como
inicialmente había 4 casillas negras en cada columna entonces no se pueden poner
todas las negras en la mitad izquierda ya que tendríamos 8 negras en cada una de las
primeras 4 columnas. ‡

8
• Inicialmente se tiene la terna (3, 4, 12). En cada paso se puede tomar los
números a y b y cambiarlos por 0,6a − 0,8b y 0,8a + 0,6b. ¿Se puede llegar
a (4, 6, 12)?

Notemos que:

(0,6a−0,8b)2 +(0,8a+0,6b)2 = 0,36a2 −0,48ab+0,64b2 +0,64a2 +0,48ab+0,36b2 = a2 +b2

Entonces la suma de los cuadrados de los 3 números nunca cambia. Es decir que si
llegamos a la terna (a, b, c), luego a2 +b2 +c2 = 32 +42 +122 = 169, pero 42 +62 +122 =
196 > 169. Entonces no se puede llegar a (4, 6, 12). ‡

A veces no es tan sencillo encontrar “la cuenta correcta“....

• Un círculo está dividido en 6 sectores (como en la figura), los números


1, 0, 1, 0, 0, 0 están escritos en ellos. En cada turno se puede sumarle 1 a
dos sectores adyacentes. ¿Es posible lograr que todos los sectores tengan
el mismo número?

Si a1 , a2 , a3 , a4 , a5 , a6 son los números escritos entonces S = a1 −a2 +a3 −a4 +a5 −a6
permanece invariante. Inicialmente S = 2, si todos los números se vuelven iguales
entonces sería S = 0. ¡Lo que es absurdo! De donde nunca los 6 sectores tendrán el
mismo número. ‡

9
• Si se tiene el par {x, y} se lo puede cambiar por el par {y, x} , {x − y, y}
o {x + y, y}. ¿Se puede obtener el par {819, 357} a partir del par {1, 2}?
¿Qué pares se puede obtener a partir del {1, 2}?

Notemos que el máximo común divisor de los 2 números que forman el par no
varía, pues (x, y) = (y, x) = (x − y, y) = (x + y, y). Como (1, 2) = 1 y (819, 357) = 21
entonces no es posible llegar de {1, 2} a {819, 357}.
Si desde {1, 2} se puede llegar a {a, b} entonces (a, b) = 1. Para ver que desde
{1, 2} se puede llegar a cualquier par de números coprimos notemos que si de un
par se puede llegar a otro, entonces desde el último se puede volver al primero (esto
se debe a que las operaciones son reversibles). Entonces tenemos que ver que desde
{a, b} se puede llegar a {1, 2}. Procedemos de la siguiente forma, dado {a, b} siempre
hagamos que el primer número sea más grande que el segundo y luego le restamos el
segundo al primero. Repetimos lo anterior hasta que obtengamos un 0. Para ver que
efectivamente esto debe ocurrir supongamos lo contrario y notemos que los números
siempre se mantienen no negativos pero en cada turno se van haciendo más y más
chicos lo que es absurdo. Si llegamos a {0, m} entonces m = (0, m) = (a, b) = 1 de
donde m = 1. Es fácil ver que desde {0, 1} se puede llegar a {1, 2}. ‡

Es muy común que si bien la suma o el producto no permanecen invariantes, sí lo


hace su paridad o su resto en la división por 3 (o por algún otro número elegido de
forma adecuada).

• En una isla hay 15 camaleones azules, 17 rojos y 19 verdes. Cuando 2


camaleones de distinto color se encuentran ambos cambian al tercer color.
¿Es posible que en algún momento todos los camaleones se vuelvan del
mismo color?

Pongámosle a cada camaleón azul un sombrero con un 1, a cada camaleón rojo


uno con un 2 y a cada camaleón verde uno con un 3. Cuando un camaleón cambia de
color, también cambia su número (al correspondiente según el nuevo color). Cada vez
que se encuentran 2 camaleones y cambian de color, la suma de los números aumenta
o disminuye en 3 o permanece igual. Entonces si bien no permanece invariante, sí lo
hace su resto en la división por 3. Inicialmente la suma es 15 + 2 × 17 + 3 × 19 =
15 + 34 + 57 = 106 que tiene resto 1 en la división por 3. Si todos los camaleones se
volvieran del mismo color, digamos al correspondiente al número k, entonces la suma
sería 51k que es múltiplo de 3 (hay 51 camaleones en total). ¡Absurdo! Entonces los
camaleones no pueden volverse todos del mismo color. ‡

10
• En un pizarrón hay cierta cantidad de letras a, b, c. En cada turno se
puede reemplazar una a y una b por una b, una a y una c por una c,
una b y una c por una a, dos a por una a, dos b por una c o dos c por
una b. El objetivo es dejar una sola letra. Probar que sin importar en qué
orden realicemos las operaciones, la letra que quede al final siempre será
la misma.

Reemplacemos a cada a por un 0, a cada b por un 1 y a cada c por un 2. Notemos


que las reglas para borrar letras y escribir otra se pueden reemplazar por la siguiente
más simple, se eligen 2 números y se los reemplaza por el resto de su suma en la
división por 3. Es fácil ver que la suma de todos los números escritos tendrá siempre
el mismo resto en la división por 3 y éste será el número que quede al final. Entonces
sin importar el orden en que se realizaron las operaciones, la última letra (o número)
será siempre la misma. ‡

• Nos es dado un tablero de 2 × 2 con números en sus casillas. En cada


paso podemos realizar una de las siguientes operaciones:
-Sumar a una fila un múltiplo de la otra.
-Sumar a una columna un múltiplo de la otra.
Si inicialmente había dos 1 en una diagonal y 0 en las otras 2 casillas. ¿Es
posible conseguir una fila de 0?

a b 1 0 ∗ ∗

c d 0 1 0 0

Para cada tablero con los números a, b, c, d en sus casillas como muestra la figura,
vamos a llamar a ad − bc su “determinante“. Es fácil ver que éste no varía con las
operaciones permitidas, pues si por ejemplo sumamos a la segunda fila k veces la
primera entonces

a(d + kb) − b(c + ka) = ad + akb − bc − bka = ad − bc

Inicialmente el determinante es 1 y si conseguimos una fila de ceros sería 0. ¡Absurdo!


Entonces no es posible conseguir una fila de ceros. ‡

11
Algunos problemas para pensar

1. Sea n un entero positivo impar y supongamos que en el pizarrón están escritos


los números 1, 2, .., 2n. En cada turno se deben elegir 2 de ellos, digamos a y b,
se los borra y se escribe |a − b|. Probar que luego de 2n − 1 pasos, el número
escrito será impar.
2. Un dragón tiene 100 cabezas. Un caballero le puede cortar 15, 17, 20 o 5 cabezas
pero al hacerlo, al dragón le crecen 24, 2, 14 y 17 cabezas nuevas respectiva-
mente. ¿Puede el caballero dejar al dragón sin cabezas?
3. Tenemos 50 peones en un tablero de 10 × 10 de forma que 25 de ellos están
en la esquina inferior izquierda de 5 × 5 del tablero y los 25 restantes están en
la esquina superior derecha de 5 × 5 (uno por casilla). En cada turno podemos
elegir un peón y hacerlo saltar por sobre cualquier otro peón vecino a la casilla
siguiente si ésta se encuentra vacía (cada peón a lo sumo tiene 8 vecinos). ¿Es
posible que luego de varios turnos los 50 peones estén en la mitad izquierda del
tablero?
4. Borramos el primer dígito de 71996 y se lo sumamos al número obtenido. Se repite
esta operación hasta obtener un número de 10 dígitos. Probar que el número de
10 dígitos obtenido tendrá 2 dígitos iguales.
5. En el pizarrón están escritos los números 1, 21 , 13 , .., n1 . En cada turno se deben
elegir 2 de ellos, digamos a y b, borrarlos y escribir en su lugar ab + a + b. ¿Qué
número queda después de n − 1 turnos?
6. Sean a1 , a2 , .., an tales que ai = ±1 y S = a1 a2 a3 a4 +a2 a3 a4 a5 +...+an a1 a2 a3 = 0.
Probar que n es múltiplo de 4.
7. En cada vértice de un hexágono hay una semilla, en cada turno se pueden
escoger 2 de ellas y moverlas 1 vértice en direcciones opuestas. Probar que no
es posible llevar todas las semillas a un solo vértice.
8. Nos es dado un tablero de 3 × 3 con números en sus casillas. En cada paso
podemos realizar una de las siguientes operaciones:
-Sumar a una fila un múltiplo de otra.
-Sumar a una columna un múltiplo de otra.

a b c
d e f
g h i

12
Probar que la siguiente cantidad permanece invariante:

a(ei − f h) − b(di − f g) + c(dh − eg)

Nota: Este número es el determinante de la matriz (tablero) anterior. Es fácil


ver que si intercambiamos 2 columnas entre sí entonces el determinante sigue
siendo el mismo pero con distinto signo. ¿Cuál será el determinante de una
matriz de n × n?

9. Tenemos un tablero de 3 × 3 con números enteros y le podemos aplicar las


mismas operaciones que en el problema anterior. Un menor de 2 × 2 del tablero
es el determinante del tablero que se obtiene al borrar una fila y una columna.
Probar que el máximo común divisor de todos los menores de 2 × 2 no varía.

13
2.3. Luces
A veces, si bien el primer invariante que se nos ocurre no funciona, lo único que
tenemos que hacer es usar la misma cuenta pero en un conjunto más chico. Veamos
algunos ejemplos.

En cada punto de la figura hay una luz. En cada turno podemos elegir una
fila o columna y cambiar el estado de todas las luces en ella. Inicialmente
todas las luces están apagadas salvo la marcada en la figura. ¿Es posible
lograr apagar todas las luces?

No, no es posible. La solución es sencilla, alcanza notar que siempre habrá una
cantidad impar de luces prendidas.
Dicho de otra forma, pongamos un 1 por cada luz apagada y un −1 por cada
luz prendida. Cada vez que cambiamos el estado de una luz estamos multiplicando
su número por −1. Como en cada turno cambiamos el estado de 4 luces entonces
multiplicamos 4 números por −1, de donde el producto de los 16 números permanece
invariante.
Si logramos apagar todas las luces el producto sería 1, lo que es imposible ya que
inicialmente era −1. ‡

¿Qué pasa si en el problema anterior también nos permitimos cambiar el


estado de cualquier diagonal de cualquier longitud? (En particular pode-
mos cambiar el estado de cualquier esquina). ¿Podemos apagar todas las
luces?

14
El argumento anterior no funciona ya que el producto no permanece invariante,
pero..... ¿Por qué multiplicar todos los números? ¿Por qué no buscar un conjunto más
pequeño cuyo producto sí permanece invariante?

Si consideramos las 8 luces marcadas de negro en la figura, cada fila, columna


o diagonal cambia el estado de 0 o 2 de estas 8 luces, luego el producto de estos 8
números debe ser “Invariante“. Como inicialmente es -1 nunca podremos apagar todas
las luces ya que esto supone llegar a un producto = 1. ‡

En cada punto de la figura hay una luz, en cada turno se puede elegir
un lado o diagonal del pentágono y cambiar el estado de todas las luces
que están en esa línea. Inicialmente todas las luces están apagadas. ¿Es
posible prender las 10 luces?

15
Como antes, reemplacemos las luces apagadas por un 1 y las prendidas por -1. Esta
vez estamos en la situación opuesta a la anterior. El producto permanece invariante
pero el inicial es el mismo al que queremos llegar, lo que no nos lleva a ningún absurdo.
La estrategia es la misma, buscar un conjunto más pequeño tal que el producto
permanezca invariante pero que sea distinto el inicial y el final.
Tomemos los 5 puntos exteriores de la figura, los vértices del pentágono. Como
cada lado o diagonal toca exactamente 2 de estos 5 puntos entonces en cada turno el
producto de estos 5 números es multiplicado por (−1)2 = 1, es decir que permanece
invariante.
Inicialmente el producto es 1, pero si conseguimos tener las 10 luces prendidas
sería −1. ¡Absurdo! Entonces no es posible lograr que todas estén prendidas. ‡

En los ejemplos anteriores vimos 2 situaciones de cómo buscar el invariante ade-


cuado cuando el más natural no nos sirve, ya sea porque no permanezca invariante o
porque no nos lleve a ningún absurdo. Veamos un ejemplo más.

Se tienen 8 luces en fila. En cada turno se pueden elegir 3 luces consecutivas


y cambiar su estado. Si inicialmente todas las luces están apagadas salvo
las 2 en los bordes, ¿es posible apagar todas las luces?

Pongamos un 1 por cada luz apagada y un -1 por cada luz prendida. La operación
permitida es multiplicar por −1 cualesquiera 3 números seguidos. Notemos que si
consideramos el producto total, éste varía y da lo mismo en la situación inicial y la
final. Es por ello que debemos buscar un conjunto más pequeño de luces.

Si consideramos las luces marcadas en la figura. Entonces en cada operación vamos


a cambiar el estado de 2 luces, que es par. Como inicialmente el producto es −1 no
podemos apagar todas ya que esto nos lleva a un producto = 1. ‡

16
Algunos problemas para pensar
1. Se tienen 16 luces ordenadas en 4 filas y 4 columnas, en cada turno se pueden
elegir 3 luces seguidas (en la misma fila o en la misma columna) y cambiar el
estado de las 3. Si inicialmente están todas prendidas... ¿Es posible apagar las
16 luces?

2. Se tienen 100 luces ordenadas en 10 filas y 10 columnas, en cada turno se pueden


elegir 4 luces seguidas (en la misma fila o en la misma columna) y cambiar su
estado. Si inicialmente están todas prendidas... ¿Es posible apagar las 100 luces?

3. (Torneo de las Ciudades 2003) En cada casilla de un tablero de 4 × 4 hay un


signo + o un signo −. La operación permitida es elegir una casilla y cambiar el
signo de esa casilla junto con todos los de sus vecinas (que tienen un lado en
común). Determinar cuántos tableros diferentes se pueden obtener.

4. (Torneo de las Ciudades 2004)


a) En cada casilla de un tablero de m × n con m, n ≥ 2 hay un signo + o un
signo −, la operación permitida es cambiar todos los signos de una fila o todos
los signos de una columna. Diremos que un tablero es irreducible si es imposible
mediante operaciones permitidas transformarlo en uno con signos + en todas
las casillas. Demostrar que todo tablero irreducible contiene un cuadrado de
2 × 2 que es un tablero irreducible de 2 × 2.
b) En cada casilla de un tablero de m × n con m, n ≥ 4 hay un signo + o
un signo −, la operación permitida es cambiar todos los signos de una fila o
todos los signos de una columna o todos los signos de una diagonal. Diremos
que un tablero es irreducible si es imposible mediante operaciones permitidas
transformarlo en uno con signos + en todas las casillas. Demostrar que todo
tablero irreducible contiene un cuadrado de 4 × 4 que es un tablero irreducible
de 4 × 4.

5. Supongamos que tenemos n lámparas y algunos interruptores. Cada interruptor


cambia el estado de algunas lámparas. Inicialmente hay algunas lámparas pren-
didas. El objetivo es apretar algunos de los interruptores para apagarlas todas.
Probar que si esto no es posible entonces se pueden elegir algunas lámparas de
forma que no importa qué interruptores apretemos, siempre habrá una cantidad
impar de lámparas prendidas.

17
2.4. Piedras
El problema que sigue fue propuesto por Konstechiv para el II Torneo de las
Ciudades en 1981. Nosotros presentamos una pequeña modificación del problema
original.

Consideremos todos los puntos del plano con coordenadas enteras no neg-
ativas. Inicialmente tenemos una piedra en (0, 0), una en (0, 1) y otra en
(1, 0). En cada paso debemos elegir una piedra en un punto (m, n) y si los
puntos (m, n + 1) y (m + 1, n) se encuentran vacíos, quitamos la piedra
de (m, n) y ponemos una en (m, n + 1) y otra en (m + 1, n).
¿Es posible que luego de una cantidad finita de pasos logremos dejar vacíos
los puntos (0, 0), (0, 1) y (1, 0)?

Lo primero que uno piensa es que debería poderse lograr el objetivo. Uno se ve
tentado a creer que podrá ir llevando cada piedra suficientemente lejos como para
que no moleste la retirada del resto. Luego de experimentar un poco caemos en que
no es tan simple. Cada vez tenemos más piedras y éstas cada vez van limitando más
los movimientos del resto. De alguna forma, las piedras más cercanas a (0, 0) son
más “pesadas“ que las más alejadas ya que les cuesta más moverse. Veamos como los
invariantes nos ayudan a formalizar esto y demostrar que es imposible limpiar los 3
puntos (0, 0), (0, 1) y (1, 0).
1
El peso de una piedra en el punto (m, n) será 2m+n . Nuestro invariante es la suma de
los pesos de todas las piedras, que llamaremos peso total. Notemos que efectivamente
esta cantidad no varía en cada paso pues:
1 1 1
= +
2m+n 2m+n+1 2m+1+n
Inicialmente el peso total es 1 + 21 + 12 = 2. Como en cada punto a lo sumo
puede haber una piedra entonces es imposible llegar a una situación con los puntos
(0, 0), (0, 1), (1, 0) vacíos ya que en ese caso el peso total sería < 2 pues:

18
1
1. Las piedras en la primera columna a lo sumo suman2 4
+ 18 + 1
16
+ .... = 1
2
1
2. Las piedras en la segunda columna a lo sumo suman 4
+ 18 + 1
16
+ .... = 1
2
1
3. Las piedras en la tercera columna a lo sumo suman 4
+ 18 + 1
16
+ .... = 1
2
1 1 1 1
4. Las piedras en la cuarta columna a lo sumo suman 8
+ 16
+ 32
+ .... = 4
1 1 1 1
5. Las piedras en la quinta columna a lo sumo suman 16
+ 32
+ 64
+ .... = 8

6. .......

Es decir que entre todas las piedras a lo sumo suman


1 1 1 1 1 1
+ + + + + + .... = 1 + 1 = 2
2 2 2 4 8 16
Pero para darse la igualdad deberíamos usar TODOS los puntos por lo que nece-
sitaríamos infinitas piedras e infinitos turnos. ¡Absurdo! De donde no es posible dejar
vacíos los puntos (0, 0), (0, 1) y (1, 0). ‡

Algunos problemas para pensar


1. Tenemos el mismo problema que antes pero con sólo 1 piedra en la casilla (0, 0).
El objetivo es dejar vacías las casillas (0, 0), (0, 1), (1, 0), (0, 2), (1, 1), (2, 0). ¿Es
posible lograrlo? (Ésta es la segunda parte del problema original, la primera era
con las 6 casillas ocupadas)

2. (Moscú 1995) Tenemos 4 triángulos rectángulos iguales. En cada turno, pode-


mos elegir cualquier triángulo de los que tenemos y cortarlo en 2 por la altura
del triángulo rectángulo (notar que quedan formados 2 triángulos rectángulos).
Probar que siempre vamos a tener 2 triángulos iguales.

3. Se tiene una tira infinita de casilleros con algunas piedras distribuidas entre ellos
(puede haber más de una piedra por casillero). En cada turno se pueden quitar
2 piedras del mismo casillero y agregar una piedra al casillero inmediatamente
a la derecha. Probar que sin importar como procedamos, llega un momento en
que no podemos seguir moviendo piedras y la distribución final es independiente
de nuestras elecciones.

2
Para sumas de este tipo consultar el apéndice “Sumas infinitas“

19
2.5. Fichas
A veces resulta necesario llevar a cabo un procedimiento complejo para elaborar
un invariante, pero el solo hecho de proponerse la construcción del mismo puede llevar
a que lo hallemos.
Lo que sigue es un caso particular de un problema de la Olimpíada Rusa de 1997.
El problema gustó tanto que fue votado como “El mejor Problema del año“ por los
participantes.

Consideremos una tira infinita de casilleros numerados con los números


enteros. En algunos casilleros hay fichas de forma que la cantidad total de
fichas es finita (puede haber casilleros con 2 o más fichas). En cada paso
se pueden realizar 2 tipos de jugadas

1. Si hay 2 casillas no vacías seguidas, n y n + 1, se saca una de cada


una y se pone una ficha en la casilla n + 2.
2. Si hay al menos 2 fichas en la casilla n, se retiran las 2 fichas y se
pone una en la casilla n + 1 y otra en la casilla n − 2.

Demostrar que toda secuencia de movidas conduce irremediablemente a


una posición desde la cual es imposible realizar nuevas movidas y sin
importar las jugadas que se hicieron siempre se llega a la misma posición.

Lo primero que haremos en este problema es buscar un invariante. La idea es


escribir en cada casilla un número que representará el peso de cada ficha en esa casilla
de forma tal que la suma de los pesos de todas las fichas (el “peso total“) no varíe al
realizar las operaciones permitidas. Veamos entonces cómo escribir los números.
Si en la “n−ésima“ casilla escribimos el número an (notar que n puede ser negativo)
y cualquier operación deja invariante el peso total, entonces en cada paso la suma de
los pesos de las fichas que retiramos debe ser lo mismo que la suma de los pesos de
las fichas que agregamos. Esto se traduce en que para cada n:

1. an+2 = an+1 + an

2. 2an = an+1 + an−2

Cuando se tienen sucesiones con este tipo de relaciones3 es común poner an = αn


para todo n y las condiciones se transforman en:

3
Mirar el apéndice “Sucesiones recurrentes“

20
1. αn+2 = αn+1 + αn

2. 2αn = αn+1 + αn−2

Sacando factor común αn en la primer ecuación y αn−2 en la segunda llegamos a:

1. α2 = α + 1

2. 2α2 = α3 + 1

La ventaja de hacer el reemplazo anterior es que de esta forma las condiciones ya


no dependen de n. Antes√ teníamos infinitas condiciones y ahora solamente dos. No es
difícil ver que α = 1+2 5 , que es√
la raíz positiva de x2 − x − 1 = 0, satisface ambas.
Pongamos entonces α = 1+2 5 , escribamos para cada n el número αn en la n−ésima
casilla y decretemos que el peso de una ficha es el número escrito en la casilla que
ocupa. Como para todo n, αn+2 = αn+1 + αn y 2αn = αn+1 + αn−2 entonces en cada
paso, sin importar qué operación realicemos, la suma de los pesos de las fichas no
varía. ¡Hemos encontrado el invariante!
Probemos que toda configuración inicial T con peso total w(T ) conduce a una
posición en donde no se pueden realizar más movimientos. Como α > 1, debe existir
k0 tal que αk > w(T ) para todo k ≥ k0 . Al ser todos los pesos positivos nunca
podremos poner una ficha en la casilla k0 o a su derecha ya que ésta sola superaría el
peso total. Podemos concluir entonces que hay una ficha que llegó más lejos hacia la
derecha que cualquier otra. Al hacerlo, esta ficha debe quedarse quieta por el resto del
juego pues si se mueve tendríamos que poner una ficha a su derecha lo que contradice
su elección. Clavemos esta ficha y repitamos el argumento con el resto (haciendo de
cuenta que la ficha clavada no existe). Continuando de esta manera, como nunca se
crean fichas, a lo sumo se descartan, tarde o temprano todas las fichas deben estar
clavadas y no se podrán mover más fichas.
Ahora debemos probar que sin importar el orden en que se hicieron las jugadas
siempre se llega a la misma posición. En caso contrario tendríamos 2 distribuciones
distintas de fichas Ti y Tj , sin 2 en la misma casilla ni 2 en casillas consecutivas, tales
que w(Ti ) = w(Tj ). Veamos que esto es imposible, para ello podemos suponer que
ninguna ficha de Ti está en la misma casilla que una de Tj , pues en ese caso quitamos
esas fichas y tenemos un contraejemplo más chico. Si las fichas de Ti están en las
casillas i1 < i2 < ... < in (con ik−1 ≤ ik − 2) y las de Tj en j1 < j2 < ... < jm (con
jk−1 ≤ jk − 2) luego4 :

w(Ti ) = αin + αin−1 + .... + αi1


4
Consultar el apéndice “Sumas infinitas“

21
w(Ti ) < αin + αin −2 + αin −4 + ....
w(Ti ) < αin (1 + α−2 + α−4 + ...)
1 α2
w(Ti ) < αin = α in
1 − α−2 (α2 − 1)
α2
w(Ti ) < αin = αin +1
α
Luego αin +1 > w(Ti ) = w(Tj ) ≥ αjm , es decir que in + 1 > jm o lo que es lo mismo
que in ≥ jm , pero de forma análoga jm ≥ in y podemos concluir que in = jm . Es decir
que Ti tiene una ficha en la misma casilla que Tj tiene otra, lo que contradice nuestra
suposición. Entonces cualesquiera 2 distribuciones desde las que no se puedan realizar
movidas con el mismo peso total deben ser iguales.
Hemos probado entonces que no importa qué las elecciones que tomemos, siem-
pre se llega a una configuración desde donde no es posible mover más fichas y esta
configuración siempre es la misma. ‡
En el problema anterior queríamos asignar números de forma tal que determinada
cuenta sencilla permaneciera invariante a las operaciones del problema; los que no
resultarían ser tan sencillos son los números que debíamos encontrar para que se
cumpliera eso, pero el hecho es que al plantear las condiciones estos aparecieron de
forma natural.

Algunos problemas para pensar


1. ¿Qué pasa si en la solución anterior en vez de poner an = αn ponemos an = Fn
donde Fn es el n−ésimo término de la sucesión de Fibonacci definida recursiva-
mente por F0 = 0, F1 = 1 y Fn+2 = Fn+1 + Fn ?

2. (Rusia 1997) Sea k ≥ 2 un número natural. Consideremos una tira infinita de


casilleros numerados con los números enteros. En algunos casilleros hay fichas
de forma que la cantidad total de fichas es finita (puede haber casilleros con 2
o más fichas). En cada paso se pueden realizar 2 tipos de jugadas

a) Si hay k casillas no vacías seguidas, n, n + 1, ..., n + k − 1, se saca una ficha


de cada una y se pone una ficha en la casilla n + k.
b) Si hay al menos 2 fichas en la casilla n, se retiran las 2 fichas y se pone
una en la casilla n + 1 y otra en la casilla n − k.

Demostrar que toda secuencia de movidas conduce irremediablemente a una


posición desde la cual es imposible realizar nuevas movidas y sin importar las
jugadas que se hicieron siempre se llega a la misma posición.

22
3. Sea {Fn } la sucesión de Fibonacci definida recursivamente por F0 = 0, F1 = 1
y Fn+2 = Fn+1 + Fn . Probar que todo numero natural n > 0 tiene una única
expresión de la forma

n = Fi1 + Fi2 + ..... + Fik

donde i1 ≥ 2 y ij+1 ≥ ij + 2 para todo j.

4. (Apmo 2006) Probar que todo entero positivo se puede escribir como la suma
finita
√ de algunas potencias distintas de el número de oro. Es decir, si α =
1+ 5
2
entonces para todo n entero positivo, existen i1 , ..., ik enteros distintos no
necesariamente positivos tales que

n = αi1 + ........ + αik

5. (Imo 1993) Construir una función f : N → N tal que f (1) = 2 y para todo
n∈N

f (f (n)) = f (n) + n
f (n) < f (n + 1)

6. Se tiene una pila de n fichas que forman una columna vertical. Esta configuración
se modifica de acuerdo a las siguientes reglas. Una ficha se puede mover si
está en el tope de una columna que contiene por lo menos dos fichas más que
la columna que está inmediatamente a la derecha (si no hay columnas a la
derecha, considerar que hay columnas con 0 fichas). En cada etapa, se elige
una de las fichas que se pueden mover (si es que hay alguna) y se coloca en
el tope de la columna que está a su derecha. Si no se puede mover ninguna
ficha la configuración se llama final. Para cada n, demostrar que no importa
qué elecciones se tomen en cada etapa, la configuración final que se obtiene es
única. Describir dicha configuración en términos de n.

23
2.6. Un poco de Geometría Proyectiva
Dado un punto o y una recta L que no pasa por o, la proyección en la recta L de
un punto a desde o es el punto a0 que se obtiene como la intersección de ao y L 5 .

Inicialmente tenemos 4 puntos alineados a0 , b0 , c0 , d0 (en ese orden) tales


que a0 b0 = b0 c0 = c0 d0 = 1. En cada turno podemos elegir una recta y
proyectar los 4 puntos en la recta desde cualquier punto, luego borramos
los 4 puntos que proyectamos y nos quedamos con los nuevos. ¿Es posible
que luego de varios turnos obtengamos 4 puntos alineados a1 , b1 , c1 , d1 (en
ese orden) tales que a1 b1 = 2b1 c1 = c1 d1 = 2?

Supongamos que es posible y lleguemos a un absurdo. Para ello vamos a demostrar


que la razón doble de los 4 puntos permanece invariante. Si tenemos 4 puntos a, b, c, d
su razón doble se denota por {ac : bd} y se define como:
ab ad ab · cd
{ac : bd} = / =
cb cd cb · ad
Consideremos ahora un punto o y proyectemos 4 puntos a, b, c, d de una recta L
en otra L0 , obteniendo a0 , b0 , c0 , d0 . Debemos probar que {ac : bd} = {a0 c0 : b0 d0 }.
Sea p la intersección de L y L0 . Usando el Teorema de Menealo en los triángulos
apa0 y cpc0 con la recta bb0 o obtenemos:

pb · a0 b0 a0 o
= (1)
ab · pb0 ao
0 0
pb · c b c0 o
= (2)
bc · pb0 co
5
Si ao//L decimos que a se fue al punto del infinito de L o que se fue al infinito en la dirección
de L.

24
Ahora volvamos a usar el Teorema de Menelao en los triángulos apa0 y cpc0 pero
con la recta dd0 o.

pd · a0 d0 a0 o
= (3)
ad · pd0 ao
0 0
pd · c d c0 o
= (4)
dc · pd0 co
Luego

pb · a0 b0 pd · c0 d0 pb · c0 b0 pd · a0 d0
=
ab · pb0 dc · pd0 bc · pb0 ad · pd0
De donde

a0 b0 · c0 d0 c0 b0 · a0 d0
=
ab · dc bc · ad
0 0 0 0
ab ·cd ab · dc
0 0 0 0
=
cb ·ad bc · ad
{ac : bd} = {a0 c0 : b0 d0 }

Por lo recién visto, si fuera posible empezar con a0 , b0 , c0 , d0 y terminar con


a1 , b1 , c1 , d1 entonces:

{a0 c0 : b0 d0 } = {a1 c1 : b1 d1 }
Pero esto es absurdo pues
a0 b0 d0 c0 1·1 1
{a0 c0 : b0 d0 } = = =
b0 c0 a0 d0 1·3 3
a1 b1 d1 c1 2·2 4
{a1 c1 : b1 d1 } = = =
b1 c1 a1 d1 1·5 5
Entonces no es posible lo que pide el enunciado. ‡

Lo anterior es un ejemplo de algo muy usado en geometría; “Las Transformaciones“.


Veamos otro ejemplo. Imaginemos un punto o y 2 planos π y π 0 . La proyección de π
sobre π 0 desde o es a cada punto p en π mandarlo a la intersección p0 de op con π 0 6 .
¿Qué preserva la proyección?
6
Si op//π 0 entonces p0 es el punto del infinito de π 0 en la dirección de L.

25
La misma cuenta que hicimos en la solución anterior demuestra que la razón
doble de puntos alineados es invariante en este tipo de proyección; pero hay otra
estructura más simple que se preserva. Notemos que si a, b, c están en π y a0 , b0 , c0 son
sus proyecciones en π 0 entonces a, b, c están alineados si y sólo si a0 , b0 , c0 lo están. Dicho
de otra forma, la proyección manda “rectas en rectas“. Más aún si P, Q, R son 3 rectas
en π y P 0 , Q0 , R0 son sus proyecciones sobre π 0 entonces P, Q, R son concurrentes si y
sólo si P 0 , Q0 , R0 lo son7 .
De ahora en adelante, cuando tengamos un objeto T en π su proyección T 0 es el
conjunto de puntos de π 0 que son las proyecciones de puntos de T . La proyección de
un círculo no es necesariamente un círculo. Lo que ocurre es que no estamos mirando
la propiedad correcta, el círculo es miembro de una familia más grande de curvas: “las
cónicas“ 8 . Los 4 miembros de esta familia son los círculos, las parábolas, las elipses
y las hipérbolas. La proyección de una cónica es una cónica. Es decir “ser un círculo“
no es invariante por proyección, pero sí lo es “ser una cónica“.
Otra propiedad invariante por proyección es “ser tangente“ a una cónica. Es decir
si tenemos una recta L y una cónica ω y L0 y ω 0 son sus proyecciones entonces L
es tangente a ω si y sólo si L0 es tangente a ω 0 . Dado un punto p en el exterior de
una cónica, ¿será posible trazar las tangentes a la cónica usando sólo una regla? Por
ejemplo para un círculo podemos proceder de la siguiente forma:
Dado un círculo ω y un punto p en su exterior tomemos 2 puntos arbi-
trarios a y b en ω y tracemos las rectas pa y pb que vuelven a cortar a ω
en c y d respectivamente. Tracemos ahora las rectas ab y cd que se cortan
en q y ad y bc que se cortan en r. Por último tracemos qr que corta a ω
en s y t. Luego ps y pt son las tangentes a ω por p.

7
¡Cuidado! si P y Q son 2 rectas paralelas, P 0 y Q0 no necesariamente lo son.
8
Cada cónica ω se puede pensar como la intersección de un cono con un plano.

26
Ahora, como la construcción entera es invariante por proyección, el mismo proced-
imiento funciona para trazar las tangentes a una cónica. Es decir, supongamos que
realizamos la construcción anterior para una cónica ω y queremos probar que pr y qr
resultan tangentes a ω. Proyectemos el dibujo a una plano π 0 de forma que la proyec-
ción ω 0 de ω resulte un círculo (¿Se les ocurre cómo hacer esto?). Ahora tenemos un
punto p0 y 2 rectas por él que cortan a ω 0 en a0 y c0 la primera y b0 y d0 la segunda.
La intersección de a0 b0 y c0 d0 es q 0 y la intersección de a0 d0 y b0 c0 es r0 . Por último, q 0 r0
corta a ω 0 en s0 y t0 . Como esto no es otra cosa que la construcción anterior aplicada
a ω 0 y p0 , resulta que p0 s0 y p0 t0 son tangentes a ω 0 , de donde proyectando de vuelta,
obtenemos que ps y pt son tangentes a ω.

¿Será posible usando sólo una regla marcar el centro de un círculo? Más
precisamente, lo que preguntamos es si existe un procedimiento que dado
cualquier círculo nos permita encontrar su centro usando sólo una regla
(como el anterior para trazar las tangentes a un círculo por un punto en
su exterior). Con la regla lo que podemos hacer es lo siguiente

1. Trazar la recta que une 2 puntos ya marcados.


2. Marcar la intersección de 2 rectas ya dibujadas.
3. Trazar una recta arbitraria por un punto dado.
4. Marcar un punto arbitrario sobre el círculo original o sobre una recta
ya dibujada.

No, no es posible. Supongamos que sí, tenemos entonces un procedimiento que


para cualquier círculo nos permite encontrar su centro usando sólo una regla como en
1,2,3 o 4.

27
Imaginemos 2 planos π y π 0 y 2 círculos ω y ω 0 de forma que ω está en π y ω 0
es la proyección de ω sobre π 0 . Apliquemos nuestro procedimiento para determinar el
centro de ω de forma que cada cosa que vamos dibujando en π la vamos proyectando
a π 0 . ¿Qué podemos decir de lo dibujado en π 0 ? Resulta que si alguien viene y observa
π 0 sin saber lo que estamos haciendo en π creerá que estamos aplicando nuestro
procedimiento para determinar el centro de ω 0 pues al proyectar lo que hacemos en π
es como si estuviéramos haciéndolo en π 0 . Por ejemplo, si tenemos a y b en π, trazamos
la recta L que los une y la proyectamos obteniendo L0 , es como si directamente
hubiéramos trazado la recta que une a0 con b0 . Si marcamos la intersección p de 2
rectas L1 y L2 y la proyectamos a π 0 obteniendo p0 es como si hubiéramos marcado
la intersección de L01 y L02 .
Como nuestra construcción sirve para cualquier círculo independientemente de
nuestras elecciones (que vienen de usar la regla en cosas como en 3 y 4) entonces
cuando encontremos el centro de ω y lo proyectemos hasta π 0 , éste será el centro de
ω, pues el dibujo en π 0 es nuestro procedimiento aplicado para determinar el centro
de ω 0 . Dicho de otra forma.. ¡Nuestro procedimiento es invariante por proyección!
El problema es que si bien nuestro procedimiento es invariante por proyección, no
lo es la propiedad “ser el centro de un círculo“. En realidad, como dijimos antes ni
siquiera “ser un círculo“ lo es, de forma que debemos tener un poco de cuidado.
Para terminar el problema lo que hacemos es elegir los planos π y π 0 y los círculos
ω y ω 0 de forma que la proyección del centro de ω no sea el centro de ω 0 9 , lo que
contradice nuestra conclusión anterior. Entonces no puede existir un procedimiento
que nos permita encontrar el centro de un círculo usando sólo una regla. ‡

La demostración anterior merece algunos comentarios. En primer lugar, el prob-


lema anterior no es un problema común. Una cosa es encontrar un procedimiento
para hacer algo pero otra dramáticamente distinta es probar que tal procedimiento
no puede existir. Ejemplos de esta situación son la cuadratura del círculo, la construc-
ción de algunos polígonos regulares con regla y compás o la resolución de ecuaciones
de grado ≥ 5. Probar que no existe ninguna fórmula para encontrar las raíces de
una ecuación de grado 5 precisó (y motivó) del desarrollo de uno de los trabajos más
profundos, bellos e importantes de la matemática: “La Teoría de Galois“. En segundo
lugar, hay algo en la demostración anterior que la hace “especial“, el problema hablaba
de algo que ocurre en el plano pero la demostración es en 3 dimensiones. Otro ejemplo
de cuan importante es no encasillar el pensamiento de uno, de no limitarlo, de dejar
volar la imaginación.

9
Mirar el problema 11

28
Algunos problemas para pensar
1. Dado un punto o, un ángulo α y una constante k consideremos la siguiente
transformación del plano en sí mismo. A cada punto p lo mandamos a p0 de
forma que pop0 = α y ppo0 o = k (a mí me gusta pensarla como que tenemos un
triángulo T y cada punto p lo mandamos a p0 de forma que pop0 sea semejante a
T ). Probar que preserva colinealidad, ángulos entre rectas y razón de segmentos.

2. Dado un punto o y una constante k, consideremos la siguiente transformación del


plano en sí mismo. A cada punto p mandémoslo a un punto p0 en la semirrecta
op tal que op×op0 = k 2 . Probar que la transformación manda círculos en círculos
(para este problema considerar las rectas como círculos de radio infinito). Probar
además que
ab · k 2
a0 b 0 =
oa · ob
Deducir que preserva razón doble.

3. Dado un cuadrilátero convexo abcd probar que los 4 vértices están en una cir-
cunferencia o en una recta si y sólo si

ab · cd + ad · bc = ac + bd

4. Dado un triángulo abc y un círculo ω que corta a ab en d y e, a bc en f y g y a


ca en h e i sean p, q y r las intersecciones de las tangentes al círculo en d y e, f
y g y en h e i. Probar que aq, br y cp son concurrentes.

5. Dado un punto o en una circunferencia ω y una recta L definimos la proyección


de ω en L como la transformación que a cada punto p en ω lo manda a p0 en L
que es la intersección de op con L. Probar que preserva razón doble.

6. Una transformación de un círculo o una recta en otro círculo o recta se dice


proyectiva si se obtiene como composición de proyecciones (por ejemplo primero
proyectamos a un círculo, luego a una recta (no necesariamente desde el mismo
punto), luego a otra recta y así). Probar que las transformaciones proyectivas
preservan razón doble.

7. Consideremos 2 planos π y π 0 y una recta L que no es paralela a ninguno de los 2


planos. A cada punto p en π lo mandamos un punto p0 en π 0 que es la intersección
de una recta paralela a L por p con π 0 (es como si proyectáramos desde un punto
en el infinito en la dirección de L). Probar que esta transformación preserva
colinealidad, concurrencia, razón de segmentos paralelos y razones de áreas.

29
8. Probar que dado un triángulo T y 3 puntos a, b y c en un plano π podemos
transformarlos mediante una transformación como la del problema anterior en
a0 , b0 , c0 en un plano π 0 de forma que a0 b0 c0 sea semejante a T .

9. Sea abcde un pentágono convexo y consideremos f = bc ∩ de, g = cd ∩ ea,


h = de ∩ ab, i = ea ∩ bc y j = ab ∩ cd. Probar que si las áreas de los triángu-
los ahi, bij, cjf, df g, egh son todas iguales entonces af, bg, ch, di, ej son concur-
rentes.
Sugerencia: Proyectar de forma que a0 c0 d0 vaya a parar a los vértices de un
pentágono regular.

10. Probar que no es posible usando sólo una regla trazar una paralela a un segmento
dado.

11. Supongamos que tenemos una esfera S y un plano π 0 tangente a S en a. Sea


o el punto de S diametralmente opuesto a a. Tomemos un plano π que pase
por el centro de S pero que no sea paralelo a π 0 y llamemos ω al círculo que
es la intersección de S con π. Por último tomemos p en π 0 de forma que op sea
perpendicular a π. Probar que ω 0 , la proyección de ω a π 0 desde o, es un círculo
de centro p.

30
2.7. Cartas
Esta vez el problema es de la lista corta de la 46 IMO del 2005.

Se tienen n cartas alineadas en una fila, cada una de ellas con un lado
blanco y el otro negro, todas con su lado blanco hacia arriba. En cada
etapa, si es posible, se elige una carta con el lado blanco hacia arriba (que
no está en el borde de la fila), se quita y se dan vuelta la carta más próxima
hacia la derecha y la carta más próxima hacia la izquierda. Demostrar que
se puede llegar a una situación en la que sólo quedan dos cartas si y sólo
si n − 1 no es divisible por 3.

En primer lugar veamos que si n − 1 no es divisible por 3 entonces es posible dejar


sólo 2 cartas. Notemos que si primero sacamos la segunda carta, luego la cuarta y por
último la tercera entonces volvemos a la situación original pero con 3 cartas menos.

BBBBB....... → N N BB....... → N BN....... → BB.......


Luego, si n − 1 no es divisible por 3, repitiendo lo anterior vamos sacando de a
3 cartas hasta llegar a BB o a BBB. Si llegamos a BB ya estamos y si llegamos a
BBB entonces quitamos la del medio y estamos en N N ; en ambos casos logramos
dejar sólo 2 cartas, como queríamos.
Para ver que si n − 1 es divisible por 3 entonces no es posible dejar sólo 2 cartas
vamos a usar un invariante. Esta vez va a ser necesario de una construcción auxiliar
para entenderlo mejor.
Vamos a pensar cada carta como una orden. Dados 3 números ordenados, las
cartas negras intercambian los 2 últimos y las blancas mueven cada número un lugar
a la derecha (es decir que el primero toma el lugar del segundo, el segundo el del
tercero y el tercero el del primero).

N (xyz) = xzy
B(xyz) = zxy
Las órdenes se leen de derecha a izquierda, por ejemplo BBN (123) = 321 pues

N (123) = 132
B(132) = 213
B(213) = 321

31
O en la notación que vamos a usar

BBN (123) = BB(N (123)) = BB(132) = B(B(132)) = B(213) = 321

No es difícil ver que tenemos las siguientes relaciones

BBB = NN (1)
BBN = NB (2)
N BB = BN (3)
N BN = BB (4)

Por ejemplo

N BN (xyz) = N B(xzy) = N (yxz) = yzx


BB(xyz) = B(zxy) = yzx
Además BBB(xyz) = N N (xyz) = xyz.
Volviendo al problema, consideremos las siguientes jugadas. Empezamos con 5
cartas blancas; primero quitamos la segunda carta (de izquierda a derecha), luego la
cuarta y por último la tercera. La situación la podemos representar como sigue:

BBBBB → N N BB → N BN → BB
¿Qué tienen en común BBBBB, N N BB, N BN y BB? Para poder responder
esta pregunta es que precisamos pensar a las cartas como órdenes que se aplican a
123. Las 4 secuencias tienen en común que pensadas como un conjunto de órdenes
aplicadas a 123 dan lo mismo, es decir:

BBBBB(123) = 231
N N BB(123) = 231
N BN (123) = 231
BB(123) = 231

Es claro ahora cuál será el invariante, hay que pensar a las cartas como órdenes
y aplicarlas (de derecha a izquierda) a 123. Para ello, debemos verificar que en cada
paso el resultado final no cambia. Supongamos por ejemplo que sacamos una carta
blanca con 2 vecinas negras (los demás casos se deducen de la misma forma de las
relaciones (1), (2) y (3)), entonces debemos ver que

...N BN...(123) = ...BB...(123)

32
Pero de (4) sabemos que N BN = BB luego

...N BN...(123) = ...N BN (...(123))


...N BN...(123) = ...(N BN (...(123)))
...N BN...(123) = ...(BB(...(123)))
...N BN...(123) = ...BB(...(123))
...N BN...(123) = ...BB...(123)
Como queríamos.
Para terminar el problema, como BBB(123) = 123, solamente hace falta notar
que si n = 3k + 1 entonces

BBBB......BBB(123) = BBBB......(123) = BBBB(123) = B(123) = 312

De donde no es posible que queden 2 cartas pues

BB(123) = B(312) = 231


N B(123) = N (312) = 321
BN (123) = B(132) = 213
N N (123) = N (132) = 123
es decir que ninguno coincide con 312. ‡

La demostración anterior se puede hacer más gráfica pensando que 123 son los
vértices de un triángulo equilátero. Cada vez que tenemos una B lo rotamos 120o en
sentido antihorario y cada vez que tenemos una N lo reflejamos por la altura vertical
(es decir que intercambiamos los 2 vértices de abajo).

33
Completemos ahora la construcción del problema, así como definimos N (xyz) =
xzy y B(xyz) = zxy podríamos haber definido otras órdenes...

I(xyz) = xyz N1 (xyz) = xzy


B1 (xyz) = zxy N2 (xyz) = zyx
B2 (xyz) = yzx N3 (xyz) = yxz
Ahora por ejemplo B1 N1 N2 (xyz) = yzx pues

B1 N1 N2 (xyz) = B1 N1 (zyx) = B1 (zxy) = yzx


De la misma forma que antes, tenemos varias relaciones, por ejemplo B1 N2 = N2 ya
que

B1 N2 (xyz) = B1 (zyx) = xzy = N1 (xyz)


Análogamente se pueden deducir las siguientes relaciones (o reglas para reemplazar
2 órdenes por 1 sola):

II = I IB1 = B1 IB2 = B2 IN1 = N1 IN2 = N2 IN3 = N3


B1 I = B1 B1 B1 = B2 B1 B2 = I B 1 N1 = N 3 B1 N2 = N1 B1 N3 = N2
B2 I = B2 B2 B1 = I B2 B2 = B1 B2 N1 = N2 B2 N2 = N3 B2 N3 = N1
N1 I = N1 N1 B1 = N2 N1 B2 = N3 N1 N1 = I N1 N2 = B1 N1 N3 = B2
N2 I = N2 N2 B1 = N3 N2 B2 = N1 N2 N1 = B2 N2 N2 = I N2 N3 = B1
N3 I = N3 N3 B1 = N1 N3 B2 = N2 N3 N1 = B1 N3 N2 = B2 N3 N3 = I
No hay dificultad ahora en resolver el siguiente problema:
Se tiene en el pizarrón una secuencia de letras. En cada turno, se pueden
elegir 2 letras, borrarlas y escribir otra de acuerdo a las siguientes reglas
AA → A AB → B AC → C AD → D AE → E AF → F
BA → B BB → C BC → A BD → F BE → D BF → E
CA → C CB → A CC → B CD → E CE → F CF → D
DA → D DB → E DC → F DD → A DE → B DF → C
EA → E EB → F EC → D ED → C EE → A EF → B
FA → F FB → D FC → E FD → B FE → C FF → A
Probar que sin importar las elecciones que tomemos, la última letra que
queda es siempre la misma.

34
Algunos problemas para pensar
1. En el problema de las cartas digamos que el peso de una carta blanca es (−1)k
donde k es la cantidad de cartas negras a su izquierda. Probar que en cada etapa,
la suma de los pesos de las cartas blancas aumenta o disminuye en una cantidad
múltiplo de 3 o se mantiene igual. Usar esto para dar otra demostración.

2. En un pizarrón hay cierta cantidad de letras e, a, b, c. En cada turno se pueden


borrar 2 letras y escribir otra en función de la siguiente tabla

e a b c
e e a b c
a a e c b
b b c e a
c c b a e

Demostrar que sin importar lo que hagamos, la última letra siempre es la misma.
Sugerencia: Poner E(wxyz) = wxyz, A(wxyz) = xwzy, B(wxyz) = yzwx y
C(wxyz) = zyxw (se puede pensar que si xyzw es un cuadrado entonces A y
B es reflejarlo por sus bases medias y que C es reflejarlo por su centro).

3. (XIV Olimpíada Rioplatense) Se desea colorear cada entero positivo con un


color utilizando la mayor cantidad posible de colores de manera que se verifique
la siguiente condición: Si, en notación decimal, el número B se puede obtener
a partir del número A, suprimiéndole a A dos dígitos iguales consecutivos (aa)
o suprimiéndole a A cuatro dígitos consecutivos que formen dos pares iguales y
consecutivas (abab), entonces A y B son del mismo color.
Por ejemplo, 8, 833 y 22811 deben ser del mismo color y también 72, 676772 y
1173329898 son del mismo color.
Determinar cuál es la mayor cantidad de colores que se puede utilizar.

35
2.8. Rompecabezas
Ahora es el turno de un juego que ya era conocido en 1870. El rompecabezas,
popularizado como “Fifteen Puzzle“, es de Noyes Chapman10 .

Se tiene un rompecabezas con forma de un tablero de 4 × 4 con 15 fichas


numeradas 1, 2, ....., 15. Siempre debe haber una casilla vacía y el resto
ocupadas por una ficha cada una. En cada turno podemos elegir una
casilla vecina a la vacía y mover su ficha hasta ésta. ¿Es posible llegar de
la posición de la izquierda a la de la derecha?

1 2 3 4 1 2 3 4
5 6 7 8 5 6 7 8
9 10 11 12 9 10 11 12
13 14 15 13 15 14

Sea P0 la posición inicial y Pn a la que queremos llegar. Supongamos que existen


posiciones intermedias P1 , ..., Pn−1 de forma que cada una se obtiene de la anterior
empujando una ficha a la casilla vacía. Es fácil ver que n debe ser par. Para ello,
pintemos las casillas de blanco y negro como las de un tablero de ajedrez y notemos
que la casilla vacía siempre cambia de color. Como P0 y Pn tienen la misma casilla
vacía entonces al final del juego, ésta debe haber cambiado de color una cantidad par
de veces o lo que es lo mismo n debe ser par.

a1 a2 a3 a4
a5 a6 a7 a8
a9 a10 a11 a12
a13 a14 a15 a16

Podemos pensar que la casilla vacía está ocupada por una ficha invisible con un
16 y en cada turno podemos intercambiar esta ficha con cualquier vecina. Definamos
para una posición P , con las fichas como en la figura anterior, su signo como:
Y
sg P = sg (aj − ai )
i<j

Recordemos que sg a = 1 si a > 0 y sg a = −1 si a < 0 (el producto nunca es cero


ya que todos los ai son distintos entre sí). Vamos a usar el signo para llegar a un
absurdo. Analicemos qué pasa con el signo después de cada turno.
10
Aunque también algunos dicen que pertenece a Sam Lloyd

36
Supongamos que Pm+1 se obtiene de Pm al intercambiar ai con ai+k (es decir que
ai = 16 o ai+k = 16, además por la forma del tablero k = 1 o k = 4). Para calcular
sg Pm+1 debemos calcular el signo de el producto de todas las diferencias de los
números en Pm+1 . Este producto es muy parecido al que debemos calcular para Pm ,
la única diferencia es que los siguientes factores se dan vuelta:

ai+1 − ai , ai+2 − ai , ...., ai+k−1 − ai , ai+k − ai , ai+k − ai+k−1 , ...., ai+k − ai+1

Como 2k − 1 factores cambiaron por su opuesto, entonces

sg Pm+1 = (−1)2k−1 sg Pm = −sg Pm

De donde, recordando que n es par

sg Pn = −sg Pn−1 = sg Pn−2 = ... = −sg P1 = sg P0

Es decir que sg Pn = sg P0 . Pero es fácil ver que sg P0 = 1 y sg Pn = −1, lo que es


absurdo que viene de suponer que era posible llegar a la posición deseada. ‡
Notemos que si bien el signo no es un invariante, éste varía de forma controlada.
Si queremos un invariante, para cada posición P pongamos I(P ) = sg P t(P ) donde
t(P ) = 1 si la casilla vacía está en una casilla negra y t(P ) = −1 si está en una blanca.

Algunos problemas para pensar


1. Supongamos que tenemos 2 posiciones para el rompecabezas del problema con
el mismo signo y la casilla vacía en el mismo lugar. Probar que se puede llegar
de una a la otra mediante las operaciones permitidas.

2. (Olimpíada Balkánica 2001) Se tiene un cubo de 3 × 3 × 3 dividido en 27 celdas


unitarias. Una de las celdas está vacía y el resto está ocupada por cubitos
unitarios numerados 1, 2, ..., 26. Cada jugada consiste en empujar un cubito
unitario a una celda vecina (que comparte una cara), si ésta se encuentra vacía.
¿Es posible que después de algunas operaciones cada cubito numerado k esté
en el lugar donde empezó el cubito 27 − k?

3. El Cubo de Rubik es un cubo de 3 × 3 × 3 dividido en 27 cubitos unitarios con


cada cara del cubo grande pintada de un color distinto. En cada turno podemos
rotar 90o una cara a elección (es decir los 9 cubitos que la componen). ¿Es
posible que después de algunos turnos volvamos a la posición original salvo por
un único cubito unitario que intercambió dos de sus caras de lugar?

37
4. (Lista Corta 40 IMO) Hay n chicos y cada uno tiene una pelota con su nombre.
En cierto momento los chicos comienzan a intercambiarse las pelotas, es decir A
se la da a B y B se la da a A. Se sabe que al final del día cada chico se fue con
su pelota y que cada par de chicos intercambiaron 1 vez la pelota exactamente.
Hallar los posibles valores de n para los que esto es posible.

5. Supongamos que tenemos n objetos, cada uno de ellos con una etiqueta. En cada
turno podemos intercambiar la etiqueta de cualesquiera 2 objetos. Demostrar
que es imposible que después de una cantidad impar de turnos cada objeto
tenga su etiqueta.

6. Dada una permutación11 π de 1, 2..., n definimos


Y Y π(j) − π(i)
sg (π) = sg (π(j) − π(i)) =
i<j i<j
j−i

Probar que sg (π1 ◦ π2 ) = sg (π1 ) sg (π2 )

7. Es dado un tablero de n×n con n impar, que está cubierto con fichas de dominó,
salvo por una esquina. Las fichas de dominó se pueden deslizar por el tablero si
la casilla nueva a ocupar está vacía. ¿A qué casillas podemos trasladar la casilla
vacía?

11
Consultar el apéndice de Permutaciones para la notación.

38
2.9. Pizarrones, Tablas y Grupos
Es momento de volver sobre algunos problemas

En un pizarrón hay 10 signos + y 5 signos −. En cada turno se debe


elegir dos de ellos, borrarlos y poner un + si eran iguales y un − si eran
distintos. Probar que después de 14 turnos quedará un signo − solamente.

En un pizarrón hay cierta cantidad de letras a, b, c. En cada turno se puede


reemplazar una a y una b por una b, una a y una c por una c, una b y una
c por una a, dos a por una a, dos b por una c o dos c por una b. El objetivo
es dejar una sola letra. Probar que sin importar en qué orden realicemos
las operaciones, la letra que quede al final siempre será la misma.

La información en los problemas anteriores se puede resumir en las siguientes


tablas

a b c
+ −
a a b c
+ + −
b b c a
− − +
c c a b

Hagamos ahora el camino inverso y reconstruyamos el problema a partir de la


tabla.

En un pizarrón hay cierta cantidad de letras e, a, b, c. En cada turno se


pueden borrar 2 letras y escribir otra en función de la siguiente tabla

e a b c
e e a b c
a a e c b
b b c e a
c c b a e

Demostrar que sin importar lo que hagamos, la última letra siempre es la


misma.

39
Si ponemos un 1 por cada e y por cada a y un −1 por cada b y por cada c
entonces estamos en las mismas condiciones que en el primer problema, pues al borrar
2 números los reemplazamos por su producto. Por ejemplo

a→ 1

b → −1
c → −1
Entonces podemos saber cuál será el último número, de donde podemos saber en
qué par está la última letra entre {e, a} y {b, c}.
De la misma forma, si ponemos un 1 por cada e y cada b y un −1 por cada a y
cada c podemos saber en qué par estará la última letra entre {e, b} y {a, c}.
Juntando la información anterior podemos saber cuál será la última letra; alcanza
notar que el primer invariante nos dice en qué columna de la figura que sigue está y
el segundo en qué fila.

e b
a c

Hemos probado entonces que sin importar lo que hagamos, la última letra siempre
es la misma. ‡

La solución anterior se puede reescribir en la siguiente forma más compacta. Reem-


placemos cada e por (1, 1), cada a por (1, −1), cada b por (−1, 1) y cada c por (−1, −1).
Entonces en cada turno borramos 2 pares y los reemplazamos por su producto (el pro-
ducto de 2 pares es coordenada a coordenada). Por ejemplo

a → ( 1, −1)
b → (−1, 1)
c → (−1, −1)
Luego el producto de todos los pares es invariante y este resultado debe ser el par
con el que se termina.
De ahora en adelante por x ∗ y entendemos la letra de la tabla en la fila de “x“ y
en la columna de “y“ y lo llamamos su ∗-producto. Por ejemplo, a ∗ b = c, b ∗ b = e y
e ∗ a = a.
Por lo recién hecho, no sólo tiene sentido ∗-multiplicar 2 letras sino cualquier
cantidad de letras. La ∗-multiplicación de un conjunto de letras es la última letra que
queda luego de aplicar el proceso del problema. Por ejemplo

a ∗ b ∗ a ∗ c = (a ∗ b) ∗ (a ∗ c) = c ∗ b = e

40
Lo anterior tiene sentido porque sin importar en qué orden pongamos los paréntesis
y realicemos las operaciones, siempre se llega al mismo resultado. En este contexto
el ∗-producto de las letras en el pizarrón es un invariante, pero no podíamos usarlo
para resolver el problema porque precisamos del problema para definirlo.
El conjunto anterior con la operación ∗ es el grupo de Klein. En la sección que
viene vamos a utilizarlo para resolver algunos problemas pero es importante destacar
que no hay nada en especial en éste, todo lo que podemos hacer con el grupo de Klein
lo podemos hacer sin él.
Cada tabla como las anteriores define una operación entre ciertos elementos. Las
tablas que vimos en esta sección eran conmutativas, es decir que x ∗ y = y ∗ x para
cualquier par de elementos x, y. Sin embargo esto no es necesario como muestra la
siguiente tabla que resume el último problema de la sección anterior.

∗ a b c d e f
a a b c d e f
b b c a f d e
c c a b e f d
d d e f a b c
e e f d c a b
f f d e b c a

Cuando la tabla no es conmutativa ya no tiene sentido ∗-multiplicar conjuntos de


letras sino secuencias (finitas) de letras. Veamos un ejemplo usando la tabla anterior
para ∗-multiplicar b ∗ d ∗ f ∗ e:

b ∗ d ∗ f ∗ e = (b ∗ d) ∗ f ∗ e = f ∗ (f ∗ e) = f ∗ c = e
b ∗ d ∗ f ∗ e = b ∗ (d ∗ f ) ∗ e = (b ∗ c) ∗ e = a ∗ e = e
b ∗ d ∗ f ∗ e = b ∗ d ∗ (f ∗ e) = b ∗ (d ∗ c) = b ∗ f = e
Sin importar en qué orden hagamos las operaciones siempre nos va a dar e, por
eso es que decimos que b ∗ d ∗ f ∗ e = e. El hecho de que no importe el orden de
las operaciones es justamente el problema antes mencionado. De forma análoga para
cada tabla podemos tratar de definir una operación. La pregunta es...... ¿Qué tablas
conducen a operaciones para las que sin importar el orden en que las realicemos
siempre dan lo mismo?

41
Algunos problemas para pensar
1. Tenemos una secuencia de letras, en cada turno podemos elegir 2 letras consec-
utivas y cambiarlas por otra de acuerdo a la siguiente tabla:

e a b c
e e a b c
a a b c d
b b c e a
c c d a e

Demostrar que sin importar lo que hagamos la última letra siempre es la misma.

2. Tenemos una secuencia de letras, en cada turno podemos elegir 2 letras consec-
utivas y cambiarlas por otra de acuerdo a una tabla. Sea x ∗ y el elemento en la
fila de la “x“ y la columna de la “y“ y supongamos que para todo a, b, c:

(a ∗ b) ∗ c = a ∗ (b ∗ c)

Probar que sin importar lo que hagamos la última letra siempre es la misma.
Nota: Esta propiedad caracteriza a las “buenas tablas“.

3. En el pizarrón están escritos los números 1, 21 , 13 , .., n1 . En cada turno se deben


elegir 2 de ellos, digamos a y b, borrarlos y escribir en su lugar ab + a + b. ¿Qué
número queda después de n − 1 turnos?

4. (Rusia 1998) Una operación a ∗ b se define en el conjunto de los números reales


tal que (a ∗ b) ∗ c = a + b + c probar que a ∗ b = a + b.

5. (Leningrado 1989) Se tiene una operación a ∗ b entre los números enteros (es
decir que a cada par de enteros a, b le asocia otro a ∗ b) tal que todo entero c es
de la forma c = a ∗ b para algún par a, b de enteros. Probar que no es posible
que (a ∗ b) ∗ c = a ∗ (b ∗ c) y a ∗ b = −b ∗ a.

42
2.10. Senku
El Senku es un juego del Siglo XV III inventado por un aristócrata francés en su
aislamiento en una celda de la Bastilla. Si bien se puede jugar en muchos tableros el
objetivo es siempre el mismo: dejar la menor cantidad posible de fichas.

Se tiene un tablero de 8 × 8 con una ficha en cada casilla salvo en las


esquinas. En cada turno debemos elegir 3 casillas formando un rectángulo
de 1 × 3 con un extremo vacío y las otras 2 casillas ocupadas. A contin-
uación sacamos la ficha de la casilla del medio y a la ficha en un extremo
la ponemos en el otro extremo.


• • → • • →

¿Es posible jugar de forma que quede una sola ficha en el tablero?

Pongamos en cada casilla del tablero un elemento del grupo de Klein de la siguiente
manera:
a b c a b c a b
c a b c a b c a
b c a b c a b c
a b c a b c a b
c a b c a b c a
b c a b c a b c
a b c a b c a b
c a b c a b c a

No es difícil ver que el ∗-producto de los elementos del grupo de Klein en casillas
ocupadas por fichas se mantiene invariante. Inicialmente el producto es

a20 ∗ b20 ∗ c20 = e ∗ e ∗ e = e

Si logramos dejar una sola ficha, ésta debe estar en una casilla con una e. Como
ninguna casilla tiene una e, esto es imposible; entonces no es posible dejar sólo una
ficha. ‡

Como dijimos antes, no es necesario usar el grupo de Klein. Usando la misma


distribución de letras que en la demostración anterior, notemos que en cada salto la

43
cantidad de letras a alterna su paridad, pues o borramos una a o agregamos una a.
De la misma forma la cantidad de letras b alterna su paridad y la cantidad de letras
c también lo hace. Como inicialmente hay 20 letras de cada una, entonces siempre
deben tener la misma paridad, es decir o hay una cantidad par de cada letra o hay
una cantidad impar de cada letra. Si logramos dejar una sola letra, entonces habrá
una cantidad impar de una letra y una cantidad par de las otras 2, lo que es absurdo.
Otra alternativa es la siguiente: Consideremos por ejemplo las siguientes distribu-
ciones de números

1 1 1 1 1 1 1 −1 −1 1 −1 −1 1 −1 −1 1
−1 1 −1 −1 1 −1 −1 1 −1 −1 1 −1 −1 1 −1 −1
−1 −1 1 −1 −1 1 −1 −1 1 −1 −1 1 −1 −1 1 −1
1 −1 −1 1 −1 −1 1 −1 −1 1 −1 −1 1 −1 −1 1
−1 1 −1 −1 1 −1 −1 1 −1 −1 1 −1 −1 1 −1 −1
−1 −1 1 −1 −1 1 −1 −1 1 −1 −1 1 −1 −1 1 −1
1 −1 −1 1 −1 −1 1 −1 −1 1 −1 −1 1 −1 −1 1
−1 1 −1 −1 1 −1 −1 1 −1 −1 1 −1 −1 1 −1 −1

Como cada rectángulo de 1 × 3 tiene dos −1 y un 1, en cada distribución el producto de


los números en las casillas con fichas debe ser invariante. Inicialmente el producto es 1 en
ambos, entonces de quedar una sola ficha ésta debe estar en una casilla con un 1 en ambos
tableros. Pero como se puede ver, no hay ninguna casilla con un 1 en ambos tableros.

La demostración anterior nos muestra como si bien a veces algunos invariantes por
separado no alcanzan, al juntar la información de ambos logramos resolver el problema. La
demostración usando el grupo de Klein no es más que usar 2 invariantes al mismo tiempo.

Algunos problemas para pensar


1. Se tiene un tablero de 4 × 4 y 15 fichas. Se deben ubicar las 15 fichas una en cada
casilla dejando una casilla vacía y luego jugar al Senku. ¿Qué casilla nos conviene
dejar vacía si queremos dejar sólo 1 ficha al final?

2. (34 Imo 1993) Sobre un tablero de ajedrez infinito se juega de la siguiente manera: Al
principio hay n2 fichas dispuestas sobre el tablero en un cuadrado de n × n de casillas
adyacentes, con una ficha en cada casilla. Cada jugada es un salto de una ficha en
dirección horizontal o vertical sobre una casilla adyacente, ocupada por otra, hasta
una no ocupada, contigua a ella. La ficha sobre la que se ha saltado se retira (como
en el Senku). Halle los valores de n para los que el juego puede terminar quedando
una única ficha en el tablero.

44
3. Se tiene un tablero como el de la figura con una ficha en cada casilla salvo en el centro.
Las fichas se comen como en el Senku. ¿Es posible dejar 1 sola ficha y que ésta quede
en la casilla marcada? ¿Y dejar una sola en el centro?

4. Supongamos que tenemos un grafo plano en donde todos los vértices tienen 3 aristas.
Probar que es posible pintar las aristas con 3 colores, de forma tal que no haya dos del
mismo color que tengan un vértice en común, si y sólo si se pueden pintar las caras
con 4 colores de forma que no haya 2 caras con un lado en común del mismo color.

45
2.11. Ranas
El siguiente juego lo propuso J.H. Conway. Se mueve como en el Senku aunque también
tiene un aire al problema “Piedras“.

En un tablero infinito hay una línea que llamaremos horizonte. En algunas


casillas por debajo del horizonte hay ranas, a lo sumo una por casilla. Las
ranas se mueven de la siguiente manera, se pueden mover en horizontal o en
vertical saltando por encima de otra rana contigua siempre que la casilla a la
que salte esté vacía, comiendo la rana sobre la que saltó. ¿Es posible para alguna
disposición inicial de las ranas que una de ellas llegue a la quinta fila por encima
del horizonte?

Supongamos que tenemos una disposición inicial para las ranas, totalmente contenida
por debajo del horizonte, que les permita llegar a una casilla en la quinta fila (a la que
llamaremos “casilla especial“). Podemos suponer que la cantidad de ranas es finita, pues las
involucradas √para que una de ellas alcance la quinta fila debe serlo.
Sea w = 5−1 2
2 , que es la raíz positiva de la ecuación x + x = 1. Notemos que 0 < w < 1
y escribamos en cada casilla una potencia de w de la siguiente forma: pongamos un 1 en la
casilla especial y en las demás casillas potencias de w como muestra la figura.

1
w2 w w2
w4 w3 w2 w3 w4
w6 w5 w4 w3 w4 w5 w6
w8 w7 w6 w5 w4 w5 w6 w7 w8
w9 w8 w7 w6 w5 w6 w7 w8 w9
w9 w8 w7 w6 w7 w8 w9
w9 w8 w7 w8 w9
w9 w8 w9

46
El peso de una rana es el número en la casilla que ocupa y naturalmente el peso total
es la suma de los pesos de todas las ranas. La situación ideal sería que en cada salto el
peso total se mantenga invariante. Si bien esto no es cierto, lo que podemos ver es que esta
cantidad nunca crece.
En efecto, supongamos que la rana sobre la que se salta tenía peso wn+1 y que no estaba
en la misma columna que la casilla especial. Entonces la rana que saltó tenía peso wn+2 y
pasó a tener peso wn o tenía peso wn y pasó a tener peso wn+2 . En ambos casos el peso total
no crece, pues la rana sobre la que se saltó se retira y como w +w2 = 1 → wn+1 +wn+2 = wn
entonces:

1. wn+1 + wn+2 ≥ wn
2. wn + wn+1 ≥ wn+2

Falta analizar qué pasa si se saltó sobre una rana en la columna de la casilla especial.
Si el salto fue vertical, el mismo argumento que antes funciona; si el salto fue horizontal es
más simple pues la rana que saltó seguirá teniendo el mismo peso pero la otra se retira.
En conclusión, el peso total nunca crece.
Ahora es momento de estimar el peso total inicial, para ello debemos sumar todos los
números debajo del horizonte. Vamos a usar12 que w3 + 2w4 + 2w5 + ... = 1 pues

w3 + 2w4 + 2w5 + .... = w3 + w4 + w4 + w5 + w5 + w6 + ....


1
w3 + 2w4 + 2w5 + .... = w2 + w3 + w4 + ... = w2 =1
1−w
Luego

1. Los números en la primer fila suman w5 + 2w6 + 2w7 + ... = w2


2. Los números en la segunda fila suman w6 + 2w7 + 2w8 + ... = w3
3. Los números en la tercer fila suman w7 + 2w8 + 2w9 + ... = w4
4. Los números en la cuarta fila suman ........
1
Entonces todos los números suman w2 + w3 + w4 + .... = w2 1−w = 1. De donde, ini-
cialmente el peso total de las ranas era < 1 (para ser 1 necesitaríamos de infinitas ranas) lo
que es absurdo pues cuando una rana llegó a la casilla especial ella sola superó el peso total.
Hemos probado entonces que no existe ninguna disposición inicial para las ranas por debajo
del horizonte que les permita llegar a la quinta fila. ‡

Algunos problemas para pensar


1. Demostrar que el mínimo número de ranas para llegar a la tercer fila sobre el horizonte
es 8 y para llegar a la cuarta 20.

12
Con respecto a las “Sumas infinitas“ consultar el apéndice

47
2.12. Monovariantes
Siguiendo el ejemplo de la sección anterior, veamos algunos casos en donde en vez de
usar algo que se mantiene cuando modificamos la situación, usamos algo que siempre varía
en forma monótona (es decir siempre crece o siempre decrece).

En una fiesta hay 600 personas y 10 habitaciones, en cada segundo una persona
se cambia de habitación de forma que pasa a una que tenía más personas que
en la que estaba. Probar que tarde o temprano todas las personas estarán en la
misma habitación.

Supongamos que una habitación tenía m personas y una de ellas se fue a otra que tenía
n, con n ≥ m. Entonces (n + 1)2 + (m − 1)2 = n2 + m2 + 2(n − m + 1) ≥ n2 + m2 + 2,
de donde la suma de los cuadrados de la cantidad de personas en cada habitación siempre
crece estrictamente. Pero si llamamos S a este número, S < 10 × 6002 , entonces a partir de
cierto momento, S deja de crecer. Entonces las personas dejan de cambiar de habitación de
donde todas están en la misma. ‡

Dado un polígono no convexo le aplicamos la siguiente operación: elegimos 2


vértices no consecutivos A y B, tales que el polígono está contenido en uno
de los 2 semiplanos que determina la recta AB y se refleja una de las partes
del polígono que une A con B por el punto medio de AB. Si aplicamos esta
operación indefinidamente, probar que tarde o temprano el polígono se vuelve
convexo.

En primer lugar, cada vez que aplicamos la operación al polígono no modificamos sus
lados ni sus direcciones, solamente cambiamos el orden; luego los posibles polígonos que
podemos obtener son finitos.

48
Por otro lado, el área del polígono siempre crece. Como los posibles polígonos son finitos
entonces en algún momento tenemos que dejar de modificarlo pues como el área crece estos
deben ser todos distintos. Pero si dejamos de modificarlo quiere decir que dejamos de aplicarle
la operación o lo que es lo mismo que obtuvimos un polígono convexo. ‡
Si en vez de reflejar por el punto medio de AB lo hacemos por la recta AB entonces
el problema sigue valiendo pero es más difícil. Se conjetura que si el polígono inicial tiene
n lados entonces 2n reflexiones como las anteriores alcanzan para transformarlo en uno
convexo.

Supongamos que tenemos una secuencia de números enteros no todos iguales


S = (a, b, c, d). En cada turno los reemplazamos por (a − b, b − c, c − d, d − a).
Probar que para todo N , el módulo de alguna coordenada se volverá más grande
que N .

Sea Sn = (an , bn , cn , dn ) la secuencia que se obtiene luego de n turnos. Usando que


an + bn + cn + dn = 0 si n ≥ 1 se puede ver que

a2n+1 + b2n+1 + c2n+1 + d2n+1 = 2(a2n + b2n + c2n + d2n ) + (an + cn )2 + (bn + dn )2

Entonces a2n+1 + b2n+1 + c2n+1 + d2n+1 ≥ 2(a2n + b2n + c2n + d2n ) de donde

a2n+1 + b2n+1 + c2n+1 + d2n+1 ≥ 2n (a21 + b21 + c21 + d21 )


Ahora si tomamos n tal que 2n (a21 + b21 + c21 + d21 ) > 4N 2 entonces al menos uno entre
(an+1 , bn+1 , cn+1 , dn+1 ) debe tener módulo más grande que N . ‡

Supongamos que tenemos una secuencia de números enteros S = (a, b, c, d). En


cada turno los reemplazamos por (|a − b|, |b − c|, |c − d|, |d − a|). Probar que
tarde o temprano llegamos a (0, 0, 0, 0).

Sea Sn = (an , bn , cn , dn ) la secuencia que se obtiene luego de n turnos y pongamos


Mi = max(ai , bi , ci , di ). Notemos que Mi+1 ≤ Mi con igualdad si y sólo si el mayor de todos
tiene un vecino que es 0. No es difícil ver que si Mi > 0 entonces Mi+4 < Mi . Como Mn ≥ 0
para todo n entonces la secuencia {Mn } debe decrecer hasta llegar a 0. Pero si Mn = 0
entonces (an , bn , cn , dn ) = (0, 0, 0, 0) como queríamos. ‡

49
Se tiene un tablero infinito con n casillas pintadas de negro. En cada turno se
puede elegir una casilla y si tiene al menos 2 de sus 4 vecinas del color opuesto
le cambiamos el color. Supongamos que luego de varios turnos hay 2006 casillas
pintadas de negro de forma que ningún par de ellas son vecinas y el resto están
de blanco. Probar que n ≥ 2006.
Sea P el perímetro de las casillas negras, esto es la cantidad de lados de casillas negras
que comparten con alguna casilla blanca (dicho de otra forma, P es la cantidad de pares de
casillas vecinas de distinto color). En cada turno P se mantiene igual o se hace más chico.
Pues por ejemplo, si pintamos una casilla blanca de negro le estamos quitando al menos 2
lados al perímetro de las negras y a lo sumo le estamos agregando 2 nuevos (recordar que
una casilla blanca se puede pintar de negro sólo si tenía al menos 2 vecinas negras).
Inicialmente P es a lo sumo 4n. En la situación final el perímetro es 4 × 2006. Luego
4n ≥ 4 × 2006 es decir que n ≥ 2006. ‡

Algunos problemas para pensar


1. (San Petersburgo 1996) Hay varios números enteros escritos en el pizarrón, en cada
turno se deben elegir 2 de ellos, digamos a y b y se los reemplaza por (a, b) y [a, b].
Demostrar que luego de cierta cantidad de turnos los números dejarán de cambiar.
2. (MOP 1998) Se tiene un 2000 − agono regular y 2001 semillas distribuidas entre sus
vértices. En cada turno se debe elegir un vértice con al menos 2 semillas, quitarlas y
poner una de ellas en un vértice vecino y la otra en el otro vértice vecino. Probar que
tarde o temprano habrá al menos 1001 vértices sin semillas.
3. Probar que los vértices de cualquier grafo se pueden pintar de blanco y negro de forma
que para cada vértice, al menos la mitad de sus vecinos tiene el color opuesto.
4. (Irán TST 2005) Se tiene una cantidad finita de puntos en el plano y un círculo. En
cada turno se calcula el baricentro de los puntos en el interior del círculo y se mueve
su centro a este punto. Probar que luego de varios turnos el círculo va a dejar de
moverse.
5. (Rumania TST 2002) Todo miembro del parlamento tiene su rating personal. El par-
lamento se divide en grupos y cada miembro tiene un rating relativo, que es el cociente
de su rating personal sobre la suma de los ratings personales de todos los miembros
del grupo. Un miembro del parlamento se puede cambiar de un grupo a otro sólo si
en el nuevo grupo su rating relativo aumenta. Cada día, solamente un miembro del
parlamento puede cambiar de grupo. Demostrar que sólo son posibles un número finito
de movimientos.
6. Se tiene un tablero de 10 × 10 con algunas casillas infectadas. En cada segundo cada
casilla con al menos 2 vecinas infectadas se infecta (2 casillas son vecinas si tienen un
lado en común). Si inicialmente había 9 casillas infectadas, ¿es posible que la infección
llegue a todo el tablero?

50
2.13. Un problema y 2 soluciones
El tercer problema de la 27 Imo de 1986 decía así:

En cada vértice de un pentágono hay escrito un número entero de forma que


la suma de todos es positiva. Si x, y, z están en vértices consecutivos con y < 0
entonces los podemos cambiar por x + y, −y, y + z. La operación anterior se
repite siempre que se pueda. Probar que en algún momento todos los números
se van a volver positivos.

Sea s la suma de los 5 números escritos, s > 0 y como x + y + z = x + y − y + y + z


entonces s se mantiene invariante con cada operación.
Para cada pentágono con los números x1 , x2 , x3 , x4 , x5 en sus vértices vamos a decir que
su “valor“ es

(x1 − x3 )2 + (x3 − x5 )2 + (x5 − x2 )2 + (x2 − x4 )2 + (x4 − x1 )2


Si vn es el valor del pentágono después de n operaciones entonces vn+1 < vn . Pues si por
ejemplo el cambio fue (x2 , x3 , x4 ) → (x2 + x3 , −x3 , x3 + x4 ) (los demás casos son análogos)
entonces

vn = (x1 − x3 )2 + (x3 − x5 )2 + (x5 − x2 )2 + (x2 − x4 )2 + (x4 − x1 )2


vn+1 = (x1 −(−x3 ))2 +(−x3 −x5 )2 +(x5 −(x2 +x3 ))2 +((x2 +x3 )−(x4 +x3 ))2 +((x3 +x4 )−x1 )2
Luego vn+1 − vn = 2sx3 < 0 (cuestión de hacer la cuenta). Ahora es fácil terminar el
problema, alcanza notar que la sucesión de enteros no negativos {vn } decrece indefinida-
mente, lo que es imposible. Entonces debe ser finita, es decir que sólo se pueden realizar una
cantidad finita de operaciones o lo que es lo mismo que después de una cantidad finita de
pasos el pentágono tendrá todos los vértices con números positivos. ‡

La solución anterior, si bien interesante y muy ingeniosa, no nos dice nada acerca de
qué ocurre en el caso en que los números no sean necesariamente enteros y nos da poca
información acerca de cuántas operaciones hay que realizar o cuál es la posición final. Veamos
otra solución que se acerca más a estas preguntas.
Pongamos x1 = x6 = x11 = ... , x2 = x7 = ... , ...., x5 = x10 = ... y consideremos el
conjunto infinito T de sumas de la forma xi + xi+1 + ... + xj con 1 ≤ i ≤ 5 y i ≤ j, por
ejemplo
x3 + x4 + x5 + x6 + x7 + x8 + x9 + x10 + x11
En cada cambio del tipo (x2 , x3 , x4 ) → (x2 + x3 , −x3 , x3 + x4 ) el conjunto T se mantiene
igual salvo que x3 cambia por −x3 . Lo que pasa es que cada suma se mantiene o se inter-
cambia con otra, por ejemplo

x3 + x4 + x5 → (−x3 ) + (x3 + x4 ) + x5 = x4 + x5

51
x4 + x5 → (x3 + x4 ) + x5 = x3 + x4 + x5
Es decir que en cada paso un elemento negativo del conjunto T se cambia a positivo
y el resto queda igual. Si bien el conjunto es infinito, como s > 0, solamente puede tener
una cantidad finita de elementos negativos. Entonces a lo sumo podemos realizar finitas
operaciones, es decir que después de una cantidad finita de cambios todos los números serán
positivos.

Algunos problemas para pensar


1. Probar que en el problema anterior, no importa qué operaciones hagamos, siempre se
tarda la misma cantidad de pasos en lograr que todos sean positivos.

2. (Olimpíada de Leningrado 1990) Son dados 20 números en un círculo. Si x, y, z son


números consecutivos, se los puede cambiar por x + y, −y, y + z. ¿Es posible cam-
biar la 20−upla (1, 2, 3, 4, 5, 6, 7, 8, 9, 10, −1, −2, −3, −4, −5, −6, −7, −8, −9, −10) por
(10, 9, 8, 7, 6, 5, 4, 3, 2, 1, −10, −9, −8, −7, −6, −5, −4, −3, −2, −1) usando estas opera-
ciones?

3. (Olimpíada de Leningrado 1990) Son dados varios números en un círculo. Si x, y, z son


números consecutivos, se los puede cambiar por x + y, −y, y + z. Probar que luego de
una cantidad finita de estas operaciones se puede conseguir que todos sean positivos
y demostrar que la posición final es única.

52
2.14. Diamante Azteca
Supongamos que tenemos un tablero de 2n × 2n cuyas filas están numeradas de abajo
hacia arriba −n, .., −1, 1, ..., n y sus columnas de izquierda a derecha −n, ..., −1, 1, .., n (notar
que no hay ni fila ni columna 0). La casilla (x, y) es la que está en la columna x y la fila
y. El Diamante Azteca de orden n es la figura formada por las casillas (x, y) tales que
|x| + |y| ≤ n + 1.
Cada vez que digamos que el Diamante está cubierto por fichas de dominó (cada una de
las cuales cubre 2 casillas que comparten un lado) estamos asumiendo implícitamente que
las fichas no se superponen ni salen fuera del Diamante.

Figura 1: Diamante Azteca de orden 4 y rotaciones permitidas

Se tiene un Diamante Azteca de orden n cubierto por fichas de dominó verticales.


En cada turno podemos rotar 90o cualquier cuadrado de 2 × 2 cubierto por
exactamente 2 dominós. Probar que la menor cantidad de rotaciones necesarias
para lograr que todos los dominós queden horizontales es n(n+1)(2n+1)
6 .

En primer lugar veamos como ir de “todos verticales“ a “todos horizontales“ y tratemos


de contar cuántas rotaciones usamos. Nos será de gran ayuda la siguiente observación:
“Supongamos que tenemos 2 dominós verticales a la misma altura separados por 2k − 2
columnas. Si el espacio entre ellos está completamente cubierto por dominós horizontales
entonces con 2k − 1 rotaciones podemos lograr que los 2k dominós estén horizontales.“

2k Columnas

53
Para ir de “todos verticales“ a “todos horizontales“ la estrategia es la siguiente: en la primer
etapa nos ocupamos de poner los dominós en las columnas −1 y 1 horizontales, en la segunda
tenemos que lograr que las columnas −2, −1, 1, 2 queden cubiertas por dominós horizontales
y el resto por dominós verticales, .... , en la k−ésima etapa tenemos que lograr que las
columnas −k, ...., k queden cubiertas por dominós horizontales y el resto por verticales.
Después de la n−ésima etapa todas las columnas estarán cubiertas por dominós horizontales
y habremos conseguido llegar a “todos horizontales“. Veamos que esto es posible y calculemos
cuántas rotaciones usamos.

Figura 2: Etapa 1 y par para la Etapa 2

Es fácil ver que en la primer etapa usamos solamente n rotaciones. En la segunda etapa,
asociemos a cada dominó vertical en la columna −2 otro en la columna 2 a la misma
altura (por ejemplo, los dominós marcados en la figura 2). Como el espacio entre ellos
está completamente cubierto por dominós horizontales entonces, por la observación, en 3
rotaciones podemos hacer que estén todos horizontales. Como hay n − 1 pares, uno por cada
dominó vertical en la columna −2, entonces en esta etapa usamos 3(n − 1) rotaciones.

Figura 3: Etapa 3 y pares de dominós a la misma altura asociados

54
Al principio de la k−ésima etapa tenemos las columnas −(k − 1), ...., k − 1 cubiertas por
dominós horizontales y el resto por dominós verticales. Hay n − k + 1 dominós verticales en
la columna −k, a cada uno de ellos lo asociamos a otro dominó vertical a la misma altura en
la columna k (como en la figura 3). Como entre ellos hay 2k − 2 columnas completamente
cubiertas por dominós horizontales entonces en 2k − 1 rotaciones podemos hacer que estén
todos horizontales. Como hay n − k + 1 pares precisamos (2k − 1)(n − k + 1) rotaciones.
Notemos que al finalizar esta etapa, las columnas −k, ..., k están completamente cubiertas
por dominós horizontales y el resto por dominós verticales.
Para saber cuantas rotaciones usamos en total debemos sumar (2k − 1)(n − k + 1) para k
entre 1 y n ya que después de n etapas habremos llegado a “todos horizontales“. La cantidad
de rotaciones usadas es13
n
X n(n + 1)(2n + 1)
(2k − 1)(n − k + 1) =
6
k=1

Veamos que en efecto la cantidad anterior es la menor posible. Para ello escribamos en
cada casilla un número como muestra la figura. El rango de un cubrimiento del Diamante
Azteca por dominós es la suma de los números en las casillas inferior izquierda de cada
dominó. ¿Qué pasa con el rango después de cada rotación?

Figura 4: El cubrimiento de la derecha tiene rango 16

En cada rotación, el rango crece o disminuye en 1. Para probar lo anterior alcanza con
analizar la figura 5. Supongamos que los dominós que rotamos tenían en su casilla inferior
izquierda uno el número a y el otro b y pasaron a tener uno el número b y el otro c. Como las
casillas en una misma diagonal que comparten un vértice tienen números consecutivos, es
decir |a − c| = 1, entonces el rango creció en 1 o disminuyó en 1 (dependiendo de si a = c + 1
o a = c − 1).

13
Consultar el apéndice “Sumas de Potencias“

55
Figura 5: Variación del rango en una rotación

Es fácil ver que el rango de “todos horizontales“ es 0 pues cada ficha aporta lo opuesto
que la ficha simétrica con respecto al eje de simetría horizontal del Diamante. Por otro lado,
calcular el rango de “todos verticales“ es ligeramente mas complicado. En la mitad izquierda
del Diamante la columna −k aporta −(n − k + 1)2 al rango y en la mitad derecha la columna
k aporta (n − k + 1)(n + k). Luego el rango de “todos verticales“ es14 :
n
X n
X n
X
(n − k + 1)(n + k) − (n − k + 1)2 = (n − k + 1)(2k − 1)
k=1 k=1 k=1
n n
X X n(n + 1)(2n + 1)
(n − k + 1)(n + k) − (n − k + 1)2 =
6
k=1 k=1

Como con cada rotación el rango a lo sumo disminuye en 1, para ir de “todos verticales“
a “todos horizontales“ precisamos al menos n(n+1)(2n+1)
6 rotaciones. ‡

No es difícil probar que para todo cubrimiento T de rango r(T ) valen las desigualdades:

n(n + 1)(2n + 1)
0 ≤ r(T ) ≤
6
Más aun, el único cubrimiento de rango nulo es “todos horizontales“ y el único cubrim-
iento de rango máximo es “todos verticales“. Por otro lado, r(T ) no apareció en la solución
anterior por casualidad. Se puede probar que la menor cantidad de rotaciones para ir de
un cubrimiento T hasta “todos horizontales“ es r(T ), su rango. Esto es remarcable, pues en
principio ni siquiera sabemos que sea cierto que se puede llegar en un número arbitrario de
rotaciones.
Notemos que la noción de “rango“ de un cubrimiento no sólo nos dice cuál es la menor
cantidad de rotaciones necesarias para que todos queden horizontales sino que nos propor-
ciona de una forma para hacerlo, alcanza con siempre elegir para rotar un cuadrado que
haga bajar el rango en 1.
Otro problema relacionado al Diamante Azteca es el de determinar, para cada n, la
cantidad de cubrimientos por fichas de dominó del Diamante Azteca de orden n. Podemos
14
Consultar el apéndice antes mencionado

56
ir más lejos todavía y preguntarnos: ¿Será posible calcular, para cada r y cada n, cuántos
cubrimientos del Diamante Azteca de orden n hay de rango r?
Antes de dar una respuesta a la pregunta anterior es preciso hacer una pequeña obser-
vación. En “todos horizontales“ no hay ningún dominó vertical y en cada rotación o paramos
2 dominó o acostamos 2 dominó, luego la cantidad de dominós verticales debe ser siempre
par. Como desde “todos horizontales“ podemos llegar a cualquier cubrimiento por rotaciones
entonces para cualquier cubrimiento la cantidad de dominós verticales es par.
Dado un cubrimiento T , por r(T ) denotamos su rango y por v(T ) la mitad de la cantidad
de dominós verticales en T . En la siguiente suma, T recorre todos los cubrimientos del
Diamante Azteca de orden n por dominós.
X
Pn (x, y) = xv(T ) y r(T )
T

El polinomio Pn (x, y) no es más que una forma de guardar información. El coeficiente


de xv y r de Pn (x, y) es la cantidad de cubrimientos con 2v dominós verticales y rango r. La
razón para guardar esta información de esta extraña manera es que Pn (x, y) es más fácil de
recordar que cada uno de sus coeficientes pues15 :
n−1
Y
Pn (x, y) = (1 + xy 2k+1 )n−k
k=0

Veamos como usar la formula anterior. Si por ejemplo deseamos saber cuántos cubrim-
ientos del Diamante Azteca de orden n hay, debemos poner x = y = 1 pues de esa forma
cada cubrimiento contribuye con 1 en la formula de Pn (x, y), luego
n−1 n−1
Y Y n(n+1)
Pn (1, 1) = (1 + 12k+1 )n−k = 2n−k = 2 2

k=0 k=0
n(n+1)
Es decir que hay 2 2 cubrimientos. Si ponemos x = 1, entonces cada cubrimiento
r
de rango r contribuye con y a Pn (1, y), luego la cantidad de cubrimientos de rango r es el
coeficiente de y r de Pn (1, y).
n−1
Y
Pn (1, y) = (1 + y 2k+1 )n−k
k=0

15
En realidad es más sutil; si por ejemplo queremos saber cuántos cubrimientos hay, alcanza con
evaluar el polinomio en un valor dado, pero si sólo tuviéramos la lista de sus coeficientes, sumarlos
podría resultar demasiado complicado. Por otro lado, a veces es más fácil obtener la expresión del
polinomio que cada uno de sus coeficientes por separado. El último problema es un ejemplo de esta
situación.

57
Por último, si ponemos y = 1 entonces cada cubrimiento con 2v dominós verticales
contribuye con xv a Pn (x, 1).
n−1
Y n(n+1)
Pn (x, 1) = (1 + x)n−k = (1 + x) 2

k=0

Algunos problemas para pensar


1. (San Petersburgo 1997) Se tiene el Diamante Azteca de orden n cubierto por fichas
de dominó. Probar que en a lo sumo n(n+1)(2n+1)
6 rotaciones podemos conseguir que
todos los dominós estén horizontales.

2. Se tiene un tablero con m filas y n columnas, con n par, cubierto por fichas de
dominó. Probar que mediante “rotaciones“ se puede hacer que todas las fichas queden
horizontales.
n(n+1)
3. Demostrar que hay exactamente 2 2 formas de cubrir al Diamante Azteca de orden
n con fichas de dominó.

4. Probar que para todo cubrimiento T de rango r(T ) de un Diamante Azteca de orden
n vale que:
n(n + 1)(2n + 1)
0 ≤ r(T ) ≤
6
Hallar los casos donde se da la igualdad.

5. Probar que desde cualquier cubrimiento de un Diamante Azteca se puede hacer una
rotación que baje en 1 el rango. Usar esto y el problema anterior para dar otra de-
mostración del problema 1.

6. Supongamos que tenemos n puntos numerados 1, 2, 3, ..., n. Un árbol T es una colección


de n − 1 segmentos entre los n puntos de forma que desde cualquier punto se pueda
llegar a cualquier otro viajando por los segmentos. En la siguiente suma, T recorre
todos los árboles en estos n puntos y para cada uno de ellos di (T ) es la cantidad de
segmentos que usan al punto i de vértice.
X d (T )−1 d (T )−1
P (x1 , ...., xn ) = x11 x22 ... xdnn (T )−1
T

Probar que P (x1 , ..., xn ) = (x1 + ... + xn )n−2 . En particular hay nn−2 árboles en n
puntos numerados. ¿Cuántos hay con d1 , d2 , ..., dn segmentos con vértices en 1, 2, .., n
respectivamente?

58
2.15. Pulgas
Para terminar, un hermoso problema de la 41 Imo del 2000. El problema es realmente
bueno.

Sea n ≥ 2 un número entero positivo. Inicialmente hay n pulgas en una recta


horizontal y no todas están en el mismo punto. Para un número real positivo λ,
definimos un salto como sigue:
Se eligen 2 pulgas situadas en puntos A y B con A a la izquierda de B. Luego
la pulga situada en A salta hasta el punto C de la recta, a la derecha de B, tal
que BC
AB = λ.
Determinar todos los valores de λ tales que para cualquier punto M de la recta
y cualesquiera posiciones iniciales de las n pulgas, existe una sucesión finita de
saltos que permite situar a todas las pulgas a la derecha de M .

En primer lugar notemos que alcanza con situar una sola pulga a la derecha de M ya
que después hacemos saltar al resto sobre esta pulga y logramos poner todas a la derecha
de M .
La primer pulga es la que está más a la derecha y la última es la que está más hacia la
izquierda (los nombres de las pulgas pueden cambiar después de cada salto). Si tenemos 2
posiciones para n pulgas, digamos en los puntos p1 , ..., pn la primera y p∗1 , ..., p∗n la segunda
con pn y p∗n las pulgas más a la derecha respectivamente, vamos a decir que la segunda
posición avanzó d con respecto a la primera si p∗n está a la derecha de pn y pn p∗n = d.
Definamos el combustible de un conjunto de pulgas como la suma de las distancias de
la primer pulga a las otras n − 1 pulgas. A continuación vamos a analizar qué pasa con
el combustible después de cada salto. Supongamos entonces que antes del i−ésimo salto el
combustible es Ci y después de éste las pulgas avanzan di .

Si la primera pulga sigue siendo la misma, es decir di = 0, es claro que el combustible


disminuye:
Ci − Ci+1 > 0 (1)

En caso contrario, supongamos que una pulga en A salta hasta C superando a la


primer pulga que está en B. Entonces di = BC y BC
BA ≤ λ (con igualdad si y sólo si la
pulga saltó sobre la primera). Entonces

Ci − Ci+1 = BA − BC − (n − 2)BC

59
BA
Ci − Ci+1 = BC( − (n − 1))
BC
1
Ci − Ci+1 ≥ di ( − (n − 1)) (2)
λ
Nos dividimos en 2 casos
1
1. Si λ < n−1 entonces λ∗ = λ1 − (n − 1) > 0. La idea es entonces que para avanzar d
deben gastar al menos dλ∗ combustible y como éste es finito no pueden avanzar tanto
como quieran. Para ser más precisos, (1) y (2) se resumen en

Ci − Ci+1 ≥ di λ∗

Pues en (1) podemos tomar di = 0, luego

C1 = C1 − C2 + C2 − C3 + ... + Ci − Ci+1 + Ci+1

C1 ≥ d1 λ∗ + d2 λ∗ + ... + di λ∗ + Ci+1
C1 ≥ (d1 + d2 + ... + di )λ∗

O lo que es lo mismo:

C1
d1 + .... + di ≤
λ∗
Pero después de los primeros “i“ saltos las pulgas avanzaron, en total, d1 + d2 + ... + di .
Es decir que las pulgas nunca podrán avanzar más que C 1
λ∗ .
1
2. Si λ ≥ n−1 entonces hacemos que en el i−ésimo salto la última pulga salte sobre la
primera (esto garantiza que se da la igualdad en (2)). Luego Ci ≤ Ci+1 ya que

1
Ci − Ci+1 = di ( − (n − 1)) ≤ 0
λ
Ci
Por otro lado di ≥ n−1 λ, pues antes del i−ésimo salto la primer pulga y la última
Ci Ci C1
estaban a distancia ≥ n−1 (por la definición de Ci ). Luego di ≥ n−1 λ ≥ n−1 λ.
C1
Como di ≥ n−1 λ para todo “i“ entonces las pulgas avanzan algo fijo en cada salto
entonces tarde o temprano superarán cualquier punto M de la recta.

Hemos probado entonces que las pulgas pueden ubicarse todas a la derecha de cualquier
1
punto M de la recta si y sólo si λ ≥ n−1 .‡

60
3. Apéndices

61
3.1. Sumas infinitas
“Los ignorantes suponen que infinitos sorteos requieren un tiempo infinito; en realidad
basta que el tiempo sea infinitamente subdivisible, como lo enseña la famosa parábola del
Certamen con la Tortuga“ J.L.Borges: La Lotería de Babilonia

Pensemos en Aquiles y la Tortuga. Luego de que Aquiles trate sin éxito de atraparla,
ésta se apiada del guerrero y decide esperarlo. Desde el momento en que la Tortuga deja
de escapar, Aquiles debe primero alcanzar el punto medio del camino que los separa. Una
vez hecho esto, debe alcanzar el punto medio de la segunda mitad del camino y luego el
punto medio de la segunda mitad de la segunda mitad del camino y así indefinidamente. Si
el camino medía 2, y como sabemos que Aquiles alcanza la tortuga, podemos decir que
1 1 1
1+ + + .... + n + .... = 2 (1)
2 4 2
Pero..... ¿Qué es lo que justifica la igualdad anterior? Después de todo, sólo sabemos
sumar finitos números. Lo que podemos hacer es ir calculando las “sumas parciales“ y tratar
de ver a qué se parecen.

S0 = 1 = 1
1
S1 = 1 + = 1, 5
2
1 1
S2 = 1 + + = 1, 75
2 4
1 1 1
S3 = 1 + + + = 1, 875
2 4 8
1 1 1 1
S4 = 1 + + + + = 1, 9375
2 4 8 16
Como se observa, las sumas parciales se van acercando cada vez más a 2. En efecto, como
para pasar de Sn a Sn+1 sumamos la mitad de lo que nos falta para llegar a 2, entonces
1
|2 − Sn+1 | = |2 − Sn |
2
Esto justifica que las sumas parciales están cada vez más cerca de 2, tan cerca como quer-
amos, y de allí que (1) tiene sentido.
Ahora tratemos de generalizar lo anterior y para x tal que |x| < 1 vamos a calcular

1 + x + x2 + .... + xn + ....

Lo primero que haremos es buscar una fórmula para las sumas parciales, esto es 1+x+...+xn

x(xn + .... + x + 1) = xn+1 + xn + ... + x

62
x(xn + .... + x + 1) = xn+1 + xn + ... + x + 1 − 1
x(xn + .... + x + 1) = xn+1 − 1 + xn + ... + x + 1
(x − 1)(xn + .... + x + 1) = xn+1 − 1
xn+1 − 1
xn + .... + x + 1 =
x−1
O lo que es lo mismo

1 − xn+1
xn + .... + x + 1 = (2)
1−x
Ahora analicemos qué pasa con (2) cuando n se va haciendo más y más grande. Como
|x| < 1, entonces xn+1 se hace cada vez más chico (tanto como queramos). Luego para
valores grandes de n, el numerador en (2) es muy parecido a 1 y como el denominador es
1
siempre el mismo, podemos decir que las sumas parciales son cada vez más parecidas a 1−x .
Pongamos entonces
1
1 + x + x2 + .... + xn + .... =
1−x
Recordemos que la fórmula anterior solamente vale para |x| < 1. Sólo para asegurarnos
que le dimos el valor correcto, calculemos que tan cerca está la n−ésima suma parcial
1
Sn = 1 + x + ... + xn de S = 1−x :
1
S − Sn = − (1 + x + ... + xn )
1−x
1 1 − xn+1
S − Sn = −
1−x 1−x
xn+1
S − Sn =
1−x
Como |x| < 1, entonces S −Sn se va haciendo cada vez más chico. Resumiendo, repitamos
las 2 fórmulas que hemos visto:

1 − xn+1
1 + x + .... + xn =
1−x
y si |x| < 1 entonces
1
1 + x + .... + xn + .... =
1−x
1
Por ejemplo, si x = 2 entonces
1 1 1 1
1+ + + .... + n + ... = 1
2 4 2 1− 2

63
1 1 1
1+ + + .... + n + ... = 2
2 4 2
1
Si x = 3 entonces:
1 1 1 1
1+ + + .... + n + ... = 1
3 9 3 1− 3
1 1 1 3
1+ + + .... + n + ... =
3 9 3 2
Por último si w es tal que w2 + w = 1 entonces
1
1 + w + w2 + ... + wn + ... =
1−w
1
1 + w + w2 + ... + wn + ... = 2
w

64
3.2. Sumas de potencias
Es conocida la historia de cómo Gauss sumó los números del 1 al 100 con una simple
cuenta. Lo primero que hizo fue escribirlos en una fila de menor a mayor. Después los volvió
a escribir en fila, debajo de la primera, pero esta vez de mayor a menor.

1 2 3 ..... 99 100
100 99 .. ..... 2 1

En cada columna los 2 números suman 101, hay 100 columnas de donde los números
deben sumar 100×101. Como cada número está escrito dos veces, la suma deseada es 100×101
2 .
De forma más general, con la misma idea podemos ver que:

n × (n + 1)
1 + 2 + .... + n = (1)
2
Otra manera de probar (1) es por inducción. La fórmula es cierta para n = 1, ahora
supongámosla cierta para n = k y veámoslo para n = k + 1.

1 + 2 + ... + (k + 1) = (1 + 2 + ... + k) + k + 1
k(k + 1)
1 + 2 + ... + (k + 1) = +k+1
2
k(k + 1) + 2(k + 1)
1 + 2 + ... + (k + 1) =
2
(k + 1)(k + 2)
1 + 2 + ... + (k + 1) =
2
Como la fórmula vale para n = 1, por lo que acabamos de probar también debe valer
para n = 2 y entonces también lo hace para n = 3 y así vemos que vale para todo n.
De la misma forma que acabamos de probar (1) podemos demostrar:

k(k + 1)(2k + 1)
12 + 22 + ... + k 2 = (2)
6
En efecto, la fórmula vale para n = 1, supongámosla cierta para n = k y veámoslo para
n=k+1

12 + 22 + ... + (k + 1)2 = (12 + 22 + ... + k 2 ) + (k + 1)2


k(k + 1)(2k + 1)
12 + 22 + ... + (k + 1)2 = + (k + 1)2
6
(k + 1)(k(2k + 1) + 6(k + 1))
12 + 22 + ... + (k + 1)2 =
6
(k + 1)(k + 2)(2k + 3)
12 + 22 + ... + (k + 1)2 =
6

65
Usando las identidades anteriores podemos probar otras más complicadas:
n
X n
X
(2k − 1)(n − k + 1) = 2kn − 2k 2 + 2k − n + k − 1
k=1 k=1
n
X n
X
(2k − 1)(n − k + 1) = (2n + 3)k − 2k 2 − (n + 1)
k=1 k=1
n
X n
X n
X
(2k − 1)(n − k + 1) = (2n + 3) k−2 k 2 − n(n + 1)
k=1 k=1 k=1
n
X n(n + 1) 2n(n + 1)(2n + 1)
(2k − 1)(n − k + 1) = (2n + 3) − − n(n + 1)
2 6
k=1
n
X 3(2n + 3)n(n + 1) − 2n(n + 1)(2n + 1) − 6n(n + 1)
(2k − 1)(n − k + 1) =
6
k=1
n
X n(n + 1)(3(2n + 3) − 2(2n + 1) − 6)
(2k − 1)(n − k + 1) =
6
k=1
n
X n(n + 1)(2n + 1)
(2k − 1)(n − k + 1) =
6
k=1

66
3.3. Sucesiones recurrentes
La sucesión de Fibonacci comienza con 0 y 1 y cada término se obtiene como la suma
de los 2 anteriores.

0, 1, 1, 2, 3, 5, 8, 13, 21, 34, 55, 89, 144, 233, 377, 610


Si llamamos Fn al n−ésimo término comenzando por F0 = 0, F1 = 1 entonces {Fn }
satisface:

Fn+2 = Fn+1 + Fn
Nuestro objetivo es dar una fórmula para Fn . Para ello primero hallemos otras sucesiones
más simples que satisfagan la misma recurrencia. Por ejemplo, busquemos sucesiones de la
forma:

1, x, x2 , x3 , ......, xn , xn+1 , xn+2 , ....


La condición de que cada término sea la suma de los 2 anteriores se traduce en que para
todo n

xn+2 = xn+1 + xn

1+ 5
Luego x2 = x + 1. Resolviendo la cuadrática llegamos a 2 posibles valores, x1 = 2 y

1− 5
x2 = 2 . Es decir que para todo n
√ √ √
1 + 5 n+2 1 + 5 n+1 1+ 5 n
( ) =( ) +( )
2 2 2
√ √ √
1 − 5 n+2 1 − 5 n+1 1− 5 n
( ) =( ) +( )
2 2 2
√ √
1+ 5 1− 5
Es fácil ver ahora que si ponemos an = αxn1 +βxn2 (recordar que x1 = 2 y x2 = 2 )
entonces

an+2 = an+1 + an
¿Será posible encontrar α y β de forma que an = Fn para todo n? ¡Sí! Por empezar si
a0 = F0 = 0 y a1 = F1 = 1 luego

α + β = a0 = 0
√ √
1+ 5 1− 5
α +β = a1 = 1
2 2
Resolviendo el sistema de ecuaciones hallamos que α = √15 y β = − √15 . Es decir que
√ √
1 1+ 5 n 1 1− 5 n
an = √ ( ) −√ ( )
5 2 5 2

67
Entonces an+2 = an+1 + an , a0 = 0 y a1 = 1. Pero como {an } y {Fn } comienzan igual
y cumplen la misma recurrencia entonces deben ser iguales. Es decir que
√ √
1 1+ 5 n 1 1− 5 n
Fn = √ ( ) −√ ( )
5 2 5 2
Hagamos otro ejemplo para ver cómo funciona esto. Definamos la sucesión {Gn } por
G0 = 6 y G1 = 13 y

Gn+2 = 5Gn+1 − 6Gn


Tratemos de hallar una fórmula para Gn . Los primeros términos de la sucesión son

6, 13, 29, 67, 161, 403, 1049, 2827, 7841, 22243, ....
Las sucesiones de la forma {xn } que satisfacen la misma recurrencia son aquellas tales
que para todo n

xn+2 = 5xn+1 − 6xn


O lo que es lo mismo

x2 = 5x − 6
Las soluciones de la ecuación anterior son x = 2 y x = 3 entonces queremos hallar α y
β tales que

Gn = α · 2n + β · 3n
Pero G0 = 6 y G1 = 13, esto implica que

α+β =6
2α + 3β = 13
Resolviendo el sistema anterior llegamos a que β = 1 y α = 5. Es decir que

Gn = 5 · 2n + 3n

68
3.4. Permutaciones
Una permutación π = (π(1), π(2), ..., π(n)) de 1, 2, ..., n es un reordenamiento de los
números 1, 2, ..., n. Por ejemplo, si n = 7 las siguientes son permutaciones de 1234567:

6437215, 1234567, 7531246, 4361275, 7654321

Para calcular cuántas permutaciones de 1, ..., n hay, alcanza notar que tenemos n opciones
para el primer lugar, luego quedan n − 1 opciones para el segundo, luego n − 2 para el
tercero,...,2 opciones para el anteúltimo y 1 opción para el último lugar. En definitiva, hay
n(n−1)(n−2).... 3 2 1 = n! permutaciones de 1, 2, ..., n. A continuación listamos las 5! = 120
permutaciones de 1, 2, 3, 4, 5.

12345 14235 21345 24135 31245 34125 41235 43125 51234 53124
12354 14253 21354 24153 31254 34152 41253 43152 51243 53142
12435 14325 21435 24315 31425 34215 41325 43215 51325 53214
12453 14352 21453 24351 31452 34251 41352 43251 51352 53241
12534 14523 21534 24513 31524 34512 41523 43512 51423 53412
12543 14532 21543 24531 31542 34521 41532 43521 51432 53421
13245 15234 23145 25134 32145 35124 42135 45123 52134 54123
13254 15243 23154 25143 32154 35142 42153 45132 52143 54132
13425 15324 23415 25314 32415 35214 42315 45213 52314 54213
13452 15342 23451 25341 32451 35241 42351 45231 52341 54231
13524 15423 23514 25413 32514 35412 42513 45312 52413 54312
13542 15432 23541 25431 32541 35421 42531 45321 52431 54321
Cuadro 1: S5 = 120 permutaciones de 1,2,3,4,5

Si tenemos una permutación π = (π(1), π(2), ..., π(n)), una “inversión“ es un par i, j tal
que i < j y π(i) > π(j). La cantidad de inversiones de π se denota por α(π) y el signo de la
permutación es sg (π) = (−1)α(π) . Por ejemplo

π = (35214) → α(π) = 6 → sg (π) = 1


π = (15342) → α(π) = 5 → sg (π) = −1
π = (14325) → α(π) = 3 → sg (π) = −1
El conjunto de permutaciones de 1, 2, ..., n se llama Sn . En realidad, Sn no es solamente un
conjunto sino un “grupo“ pues se puede operar con las permutaciones de forma natural. Para
ello tomemos n letras ordenadas y consideremos cada permutación π = (π(1), π(2), ..., π(n) )
como una orden que reordena las n letras de forma que la primer letra quede en el lugar

69
π(1), la segunda letra en el lugar π(2), la tercera en π(3),...., la última en π(n). Por ejemplo
si n = 5.

(13425)(ABCDE) = ADBCE
(21345)(ABCDE) = BACED
(51234)(ABCDE) = BCDEA
Cuando aplicamos más de una orden, éstas se leen de derecha a izquierda, por ejemplo:

(12534)(43521)(54321)(ABCDE) = (12534)(43521)(EDCBA)
(12534)(43521)(54321)(ABCDE) = (12534)(ABDEC)
(12534)(43521)(54321)(ABCDE) = (ABECD)
Es fácil deducir “reglas para reemplazar“ 2 permutaciones por otra. Por ejemplo:

(12453)(21345)(ABCDE) = (12453)(BACED) = BADCE = (21435)(ABCDE)

(21345)(12453)(ABCDE) = (21345)(ABECD) = BAECD = (21453)(ABCDE)


Luego (12453)(21345) = (21435) y (21345)(12453) = (21453). La ventaja de pensar las
permutaciones como órdenes es que podemos operar con ellas; si π1 , π2 son dos permutaciónes
entonces π2 π1 es la permutación que ordena las letras como lo hace primero aplicar π1 y
después π2 .

70
3.5. Menelao
Supongamos que tenemos un triángulo abc y puntos x, y, z en ab, bc, ca respectivamente.
¿Cómo saber si xyz están alineados?

Teorema de Menelao: Sea abc un triángulo y L una recta que corta a los lados ab, bc, ca
en x, y, z respectivamente. Luego

ax by cz
=1
bx cy az
Demostración: Sean ha , hb , hc los pies de las perpendiculares desde a, b, c a la recta L.
Luego los triángulos aha x y bhb x son semejantes de donde

ax aha
= (1)
bx bhb
Análogamente

by bhb
= (2)
cy chc
cz chc
= (3)
az aha

71
Multiplicando (1), (2) y (3):

ax by cz aha bhb chc


=
bx cy az bhb chc aha
ax by cz
=1
bx cy az
Como queríamos. ‡

Ahora veamos el “Dual“ de Menelao, el Teorema de Ceva.

Teorema de Ceva: Sea abc un triángulo y x, y, z puntos en los lados ab, bc, ca respec-
tivamente tales que cx, ay, bz se cortan en un punto p. Entonces

ax by cz
=1
bx cy az

Demostración: Recordemos que si tenemos 2 triángulos con la misma altura, la razón


de sus áreas es la razón de sus bases. Luego

by (aby) (bpy) (aby) − (bpy) (abp)


= = = =
yc (ayc) (cpy) (ayc) − (cpy) (cap)
by (bap)
=
yc (cap)
Análogamente
cz (cpb)
=
za (apb)
ax (apc)
=
xb (bpc)
De donde

72
ax by cz (apc) (bap) (cpb)
=
bx cy az (bpc) (cap) (apb)
ax by cz
=1
bx cy az
Como queríamos. ‡

Es fácil ver que valen los recíprocos de Menelao y Ceva aunque con un pequeño detalle,
como no usamos segmentos dirigidos vamos a decirlo de la siguiente manera.

“Sea abc un triángulo y x, y, z puntos en los lados ab, bc, ca (o en sus prolonga-
ciones) respectivamente. Supongamos que

ax by cz
=1
bx cy az

Entonces xyz están alineados o ay, bz, cx se cortan en un punto 16 .“

Hagamos una observación más. Supongamos que tenemos un triángulo abc y puntos
x, y, z puntos en los lados ab, bc, ca respectivamente tales que cx, ay, bz se cortan en un
punto p. Marquemos ahora y 0 la intersección de la recta xz con bc. Luego por el Teorema
de Ceva y el Teorema de Menelao:

ax by cz
=1
bx cy az
ax by 0 cz
=1
bx cy 0 az

16
Si hubiéramos usado segmentos dirigidos entonces se cumple que están alineados si el producto
es −1 y son concurrentes si el producto es 1

73
De donde

by by 0
= 0
cy cy
Es decir que17 {bc : yy 0 } = 1. Tales puntos se dicen “conjugados harmónicos“ y es una
propiedad invariante por proyección (pues de hecho la razón doble lo es). Los conjugados
harmónicos tienen la siguiente propiedad:

“Sean a, b, c, d cuatro puntos distintos en una recta conjugados harmónicos, es


decir tales que {ac : bd} = 1. Tomemos p fuera de la recta y cualquier punto q
en la recta bp. Luego sea r la intersección de cq con ap y s la intersección de aq
con cp, entonces r, s, d están alineados.“

17
Para la definción de la razón doble ir a “Un poco de Geometría Proyectiva“.

74
4. Problemas

75
4.1. Algunos problemas para pensar
1. Los números 1, 2, 3, ..., 30 están escritos en un círculo en un orden arbitrario. En cada
turno se puede intercambiar 2 números consecutivos. Si al final cada número ocupa la
posición diametralmente opuesta a la inicial. Probar que en algún momento 2 números
con suma 31 fueron intercambiados.

2. Se dispone de un tablero de una sola fila pero infinitas casillas y de un número finito de
porotos que se ubican ocupando casillas del tablero. Se realiza una sucesión de movidas
como sigue: en cada paso, se elige una casilla que tenga más de un poroto; se toman dos
de esos porotos y se colocan uno en la casilla que está inmediatamente a la derecha
y el otro en la casilla que está inmediatamente a la izquierda de la casilla elegida.
La sucesión de movidas finaliza si en algún momento hay a lo sumo un poroto en
cada casilla. Dada una configuración inicial, demostrar que toda sucesión de movidas
finalizará después de la misma cantidad de pasos y con la misma configuración.

3. (San Petersburgo 1997) El número 999..,999 (1997 nueves) está escrito en el pizarrón.
Cada minuto uno de los números del pizarrón se factoriza como producto de 2 números,
se borra el número y se reemplaza por sus 2 factores aumentados o disminuidos en
2 (independientemente). ¿Es posible que en algún momento todos los números del
pizarrón sean iguales a 9?

4. (Moscú 2002) Tenemos un collar con pn perlas (p primo y n ≥ 2). En cada operación el
collar se corta en varios pedazos del mismo largo, se da vuelta el orden de las perlas en
cada pedazo y se rearma el collar con los pedazos en el mismo orden en que estaban.
¿Se puede obtener cualquier orden de las perlas mediante tales operaciones?

5. (Torneo de Las Ciudades 1980) Se tienen algunos puntos alrededor de un círculo cada
uno de ellos coloreados de blanco o negro. Se pueden realizar 2 tipos de operaciones:

a) Se cambian de color 2 puntos consecutivos y se agrega un punto blanco entre


ambos.
b) Se quita un punto blanco y se cambia el color de los 2 vecinos.

Probar que si inicialmente sólo hay 2 puntos blancos no se puede conseguir tener sólo
2 puntos negros.

6. (Torneo de Las Ciudades 1983) Se tiene un tablero de ajedrez infinito y en cada casilla
hay un rey salvo por un conjunto de casillas C. Una movida consiste en mover algunos
reyes (posiblemente todos) a una casilla vecina (cada casilla tiene 8 vecinas) de forma
que en cada casilla quede a lo sumo 1 rey.

a) Si C es el conjunto de casillas (m, n) con m y n múltiplos de 100. ¿Es posible


que luego de una cantidad finita de movidas cada casilla tenga un rey?

76
b) Si C es el conjunto de casillas ocupadas o atacadas por un conjunto arbitrario
de damas en el tablero. ¿Es posible que luego de una cantidad finita de movidas
cada casilla tenga un rey?

7. (Torneo de Las Ciudades 1987) Se tiene un triángulo equilátero de lado n dividido


en n2 triángulos equiláteros de lado 1. Inicialmente hay un triángulo de blanco y el
resto de negro. En cada turno se puede cambiar el color de todos los triángulos entre
2 paralelas consecutivas a un lado del triángulo. ¿Puede conseguirse que todos los
triángulos sean negros?

8. (Torneo de Las Ciudades 1993) Hay 4 ranas. En cada turno una rana salta. Si la rana
que salta estaba en A y las otras en B,C y D entonces la rana salta hasta E tal que
el punto medio de AE sea el baricentro de BCD. Si inicialmente las ranas estaban en
los vértices de un cuadrado, ¿es posible que luego de algunos turnos una rana salte
encima de otra?

9. (Torneo de Las Ciudades 2005) Inicialmente no hay ningún número en el pizarrón,


en cada etapa se puede o bien escribir dos 1’s o bien borrar dos números idénticos,
digamos n y al mismo tiempo escribir los dos números n + 1 y n − 1. Determinar
el número mínimo de etapas necesarias para que al menos uno de los números del
pizarrón sea el 2005.

10. (Leningrado 1988) Se tiene una pila con 1001 piedras. En cada turno se elige una pila,
se le quita una piedra y el resto se divide en 2 pilas nuevas. ¿Es posible lograr que
todas las pilas tengan 3 piedras?

11. (Leningrado 1988) Cualquier secuencia de 00 s y 10 s es una palabra. Un triplicado es


tomar una palabra y repetirla 3 veces (una a continuación de la otra). Por ejem-
plo 010101, 111, 100110011001. Dada una palabra se la puede transformar en otra
introduciéndole o borrándole un triplicado en cualquier lugar. ¿Es posible cambiar la
palabra 01 en 10 por una secuencia de transformaciones como las recién descripta?

12. (Leningrado 1991) Se tienen varios números enteros escritos en un círculo. En cada
turno se puede quitar un número par y sumar los 2 números que eran vecinos a éste
(.., x, 2y, z, ... → ..., x + y, ...). La operación se repite hasta que queden 1 o 2 números
o que todos los números sean impares. Probar que la cantidad de números que queden
es independiente de lo que hagamos.

13. (Bamo 2006) Se tienen k manijas alineadas, cada una de los cuales está hacia arriba,
abajo, derecha o izquierda. Cuando cualesquiera 3 manijas consecutivas apunten en
direcciones distintas se pueden girar las 3 de forma que apunten a la cuarta dirección.
Probar que esta operación no se puede repetir indefinidamente.

14. (Estonia 2002) Sea n un entero positivo dado. Pongamos una ficha en cada número
negativo. En cada turno podemos elegir n enteros consecutivos, quitar una ficha de

77
uno ellos y reacomodar las demás fichas de estos n enteros como queramos en los n
enteros (a lo sumo una ficha por número). Decidir si existe n de forma que para cada
N > 0 es posible poner una ficha a la derecha de N .

15. Nos es dado un tablero de n × n con números enteros en sus casillas. En cada paso
podemos realizar una de las siguientes operaciones:
-Sumar a una fila un múltiplo de otra
-Sumar a una columna un múltiplo de otra
-Intercambiar 2 filas
-Intercambiar 2 columnas
Probar que se puede llevar al tablero a uno con los números (d1 , ..., dn ) en su diagonal
y 0 fuera de ella de forma que d1 |d2 , d2 |d3 ,... ,dn−1 |dn . Probar además que los números
d1 , ..., dn son únicos salvo por su signo (esta es la forma normal de Smith).

16. (Tercer problema de Hilbert) ¿Es posible cortar un cubo en una cantidad finita de
poliedros y con ellos armar un tetraedro?

78

Das könnte Ihnen auch gefallen